Формула сопротивления физика: Формула сопротивления

Содержание

404 page not found | Fluke

Talk to a Fluke sales expert

Связаться с Fluke по вопросам обслуживания, технической поддержки и другим вопросам»

What is your favorite color?

Имя *

Фамилия *

Электронная почта *

FörКомпанияetag *

Номер телефона *

Страна * United States (Estados Unidos)CanadaAfghanistanAlbaniaAlgeriaAmerican SamoaAndorraAngolaAnguillaAntarticaAntigua and BarbudaArgentinaArmeniaArubaAustraliaAzerbaijanBahamasBahrainBangladeshBarbadosБеларусь (Belarus)Belgien/Belgique (Belgium)BelizeBeninBermudaBhutanBoliviaBonaireBosnia and HerzegovinaBouvet IslandBotswanaBrasil (Brazil)British Indian Ocean TerritoryBrunei DarussalamBulgariaBurkina FasoBurundiCambodiaCameroonCape VerdeCayman IslandsCentral African RepublicČeská republika (Czech Republic)ChadChile中国 (China)Christmas IslandCittà Di VaticanCocos (Keeling) IslandsCook IslandsColombiaComorosCongoThe Democratic Republic of CongoCosta RicaCroatiaCyprusCôte D’IvoireDanmark (Denmark)Deutschland (Germany)DjiboutiDominicaEcuadorEgyptEl SalvadorEquatorial GuineaEritreaEspaña (Spain)EstoniaEthiopiaFaroese FøroyarFijiFranceFrench Southern TerritoriesFrench GuianaGabonGambiaGeorgiaGhanaGilbralterGreeceGreenlandGrenadaGuatemalaGuadeloupeGuam (USA)GuineaGuinea-BissauGuyanaHaitiHeard Island and McDonald IslandsHondurasHong KongHungaryIcelandIndiaIndonesiaIraqIrelandIsraelIslas MalvinasItalia (Italy)Jamaica日本 (Japan)JordanKazakhstanKenyaKiribati대한민국 (Korea Republic of)KuwaitKyrgyzstanLaosLatviaLebanonLesothoLiberiaLibyaLiechtensteinLithuaniaLuxembourgMacaoMacedoniaMadagascarMalawiMalaysiaMaldivesMaliMaltaMarshall IslandsMartiniqueMauritaniaMauritiusMayotteMéxico (Mexico)MicronesiaMoldovaMonacoMongoliaMontenegroMonserratMoroccoMozambiqueMyanmarNamibiaNauruNederland (Netherlands)Netherlands AntillesNepalNew CaledoniaNew ZealandNicaraguaNigerNigeriaNiueNorge (Norway)Norfolk IslandNorthern Mariana IslandsOmanÖsterreich (Austria)PakistanPalauPalestinePanamaPapua New GuineaParaguayPerú (Peru)PhilippinesPitcairn IslandPuerto RicoРоссия (Russia)Polska (Poland)Polynesia (French)PortugalQatarRepública Dominicana (Dominican Republic)RéunionRomânia (Romania)RwandaSaint HelenaSaint Pierre and MiquelonSaint Kitts and NevisSaint LuciaSaint Vincent and The GrenadinesSan MarinoSao Tome and PrincipeSaudi ArabiaSchweiz (Switzerland)SenegalSerbiaSeychellesSierra LeoneSingaporeSlovakiaSloveniaSolomon IslandsSomaliaSouth AfricaSouth Georgia and The South Sandwich IslandsSouth SudanSri LankaSudanSuomi (Finland)SurinameSvalbard and Jan MayenSverige (Sweden)SwazilandTaiwanTajikistanTanzaniaThailandTimor-LesteTokelauTogoTongaTrinidad and TobagoTunisiaTürkiye (Turkey)TurkmenistanTurks and Caicos IslandsTuvaluUgandaUkraineUnited Arab EmiratesUnited KingdomUnited States Minor Outlying IslandsUruguayUzbekistanVanuatuVirgin Islands (British)Virgin Islands (USA)VenezuelaVietnamWallis and FutunaWestern SaharaWestern SamoaYemenZambiaZimbabwe

Почтовый индекс *

Интересующие приборы

iGLastMSCRMCampaignID

?Отмечая галочкой этот пункт, я даю свое согласие на получение маркетинговых материалов и специальных предложений по электронной почте от Fluke Electronics Corporation, действующей от лица компании Fluke Industrial или ее партнеров в соответствии с политикой конфиденциальности.

consentLanguage

Политика конфиденциальности

По какой формуле определяется сопротивление отдельного проводника? Выберите правильный ответ,

Свет обладает корпускулярными и волновыми свойствами. Какие утверждения неверны? а) чернозем поглощает энергию света б) шероховатая поверхность рассеи … вает свет в) зеркало отражает световые лучи г) радуга есть проявлении дисперсии — разложение белого в спектр д) экран калькулятора поглощает свет

помогите пожалуйста ​

1. Если представить человека, как модель теплового устройства, то один из тепловых процессов следует исключить : а) Плавление жирной пищи в пищеварите … льном тракте б) Нагревание воздуха при прохождении через органы дыхания

РОЗПИСАТИ ДАНО! Скільки водяної пари при 100 0С треба впустити в калориметр, у якому є 200 г води при 0 0С, щоб вода нагрілася до 80 0С?

РОЗПИСАТИ ДАНО!Скільки тепла потрібно для плавлення 2 кг льоду,що має температуру 0о С, та нагрівання отриманої води до кипіння? Питома теплота плавле … ння льоду 330 кДж/кг, питома теплоємність води 4200 Дж/кг∙ оС.

А) 4,75 МДж; Б) 0,84 МДж;В) 0,66 МДж; Г) 1,5 МДж.

Знайти масу води, для нагрівання якої на 40 ºС витрачено 42000 Дж теплоти? А) 250 г; Б) 1кг 50 г; В) 10 кг; Г) 4 кг.

физика 8 класс генденштейн​

физика 8 класс генденштейн​

При каких условиях тело перебывает в состоянии покоя? Когда тело двигается неравномерно. Когда на тело не действуют другие тела или действия других те … л скомпенсированы. Когда тело не двигается. Когда на тело действуют другие тела. Вопрос №2 ? 2 балла Судно заходит в порт. Капитан отдает команду «Стоп машина», но судно продолжает двигаться. Какое физическое явление при этом наблюдается? Скольжение Столкновение Давление Инерция Вопрос №3 ? 2 балла Водитель автобуса делает резкий поворот влево, чтобы объехать препятствие. В какую сторону отклонятся пассажиры автобуса? влево вправо вперед назад Вопрос №4 ? 3 балла Футбольным мячом случайно попали в окно, и оно разбилось. При этом … только мяч действовал на окно только окно действовало на мяч окно действовало на мяч, а мяч на окно мяч и окно не взаимодействовали Вопрос №5 ? 3 балла Вы путешествуете ночью современным автобусом.

За окнами салона сплошная темнота. По каким признакам можно понять, что автобус трогается с остановки? Шума двигателя не слышно. Когда автобус трогается с остановки, видно как двигаются колёса. Когда автобус трогается с остановки, ваше тело отклоняется назад. Когда автобус трогается с остановки, ваше тело отклоняется вперед. Когда автобус трогается с остановки, слышен шум мотора.

заполните таблицу плиз​

формула расчета удельного сопротивления и закон Ома

Закон Ома является основным законом электрических цепей. При этом он позволяет объяснять многие явления природы. Например, можно понять, почему электричество не «бьет» птиц, которые сидят на проводах. Для физики закон Ома является крайне значимым. Без его знания невозможно было бы создавать стабильно работающие электрические цепи или вовсе не было бы электроники.

Зависимость I = I(U) и ее значение

История открытия сопротивления материалов напрямую связана с вольт-амперной характеристикой. Что это такое? Возьмем цепь с постоянным электрическим током и рассмотрим любой ее элемент: лампу, газовую трубку, металлический проводник, колбу электролита и т. д.

Меняя напряжение U (часто обозначается как V), подаваемое на рассматриваемый элемент, будем отслеживать изменение силы тока (I), проходящего через него. Как итог, мы получим зависимость вида I = I (U), которая носит название «вольт-амперная характеристика элемента» и является прямым показателем его электрических свойств.

Вольт-амперная характеристика может выглядеть по-разному для различных элементов. Самый простой ее вид получается при рассмотрении металлического проводника, что и сделал Георг Ом(1789 — 1854).

Вольт-амперная характеристика — это линейная зависимость. Поэтому ее графиком служит прямая линия.

Закон в простой форме

Исследования Ома по изучению вольт-амперных характеристик проводников показали, что сила тока внутри металлического проводника пропорциональна разности потенциалов на его концах (I ~ U) и обратно пропорциональна некоему коэффициенту, то есть I ~ 1/R. Этот коэффициент стал называться «сопротивление проводника», а единица измерения электрического сопротивления — Ом или В/А.

Стоит отметить еще вот что. Закон Ома часто используется для расчета сопротивления в цепях.

Формулировка закона

Закон Ома говорит, что сила тока (I) отдельно взятого участка цепи пропорциональна напряжению на этом участке и обратно пропорциональна его сопротивлению.

Следует заметить, что в таком виде закон остается верным только для однородного участка цепи. Однородной называется та часть электрической цепи, которая не содержит источника тока. Как пользоваться законом Ома в неоднородной цепи, будет рассмотрено ниже.

Позже опытным путем было установлено, что закон остается справедливым и для растворов электролитов в электрической цепи.

Физический смысл сопротивления

Сопротивление — это свойство материалов, веществ или сред препятствовать прохождению электрического тока. Количественно сопротивление в 1 Ом означает, что в проводнике при напряжении 1 В на его концах способен проходить электрический ток силой 1 А.

Удельное электрическое сопротивление

Экспериментальным методом было установлено, что сопротивление электрического тока проводника зависит от его размеров: длина, ширина, высота. А также от его формы (сфера, цилиндр) и материала, из которого он сделан. Таким образом, формула удельного сопротивления, например, однородного цилиндрического проводника будет: R = р*l/S.

Если в этой формуле положить s = 1 м2 и l = 1 м, то R численно будет равен р. Отсюда вычисляется единица измерения для коэффициента удельного сопротивления проводника в СИ — это Ом*м.

В формуле удельного сопротивления р — это коэффициент сопротивления, определяемый химическими свойствами материала, из которого изготовлен проводник.

Для рассмотрения дифференциальной формы закона Ома, необходимо рассмотреть еще несколько понятий.

Плотность тока

Как известно, электрический ток — это строго упорядоченное движение любых заряженных частиц. Например, в металлах носителями тока выступают электроны, а в проводящих газах — ионы.

Возьмем тривиальный случай, когда все носители тока однородны — металлический проводник. Мысленно выделим в этом проводнике бесконечно малый объем и обозначим через u среднюю (дрейфовую, упорядоченную) скорость электронов во взятом объеме. Далее пусть n обозначает концентрацию носителей тока в единице объема.

Теперь проведем бесконечно малую площадь dS перпендикулярно вектору u и построим вдоль скорости бесконечно малый цилиндр с высотой u*dt, где dt — обозначает время, за которое все носители скорости тока, содержавшиеся в рассматриваемом объеме, пройдут сквозь площадку dS.

При этом электронами сквозь площадку будет перенесен заряд, равный q = n*e*u*dS*dt, где e — заряд электрона. Таким образом, плотность электрического тока — это вектор j = n*e*u, обозначающий количество заряда, переносимого в единицу времени через единицу площади.

Один из плюсов дифференциального определения закона Ома заключается в том, что часто можно обойтись без расчета сопротивления.

Электрический заряд. Напряженность электрического поля

Напряженность поля наряду с электрическим зарядом является фундаментальным параметром в теории электричества. При этом количественное представление о них можно получить из простых опытов, доступных школьникам.

Для простоты рассуждений будем рассматривать электростатическое поле. Это электрическое поле, которое не изменяется со временем. Такое поле может быть создано неподвижными электрическими зарядами.

Также для наших целей необходим пробный заряд. В его качестве будем использовать заряженное тело — настолько малое, что оно не способно вызывать какие-либо возмущения (перераспределение зарядов) в окружающих объектах.

Рассмотрим поочередно два взятых пробных заряда, последовательно помещенных в одну точку пространства, находящуюся под воздействием электростатического поля. Получается, что заряды будут подвергаться неизменному во времени воздействию с его стороны. Пусть F1 и F2 — это силы, воздействующие на заряды.

В результате обобщения опытных данных было установлено, что силы F1 и F2 направлены либо в одну, либо в противоположные стороны, а их отношение F1/F2 является независимым от точки пространства, куда были поочередно помещены пробные заряды. Следовательно, отношение F1/F2 является характеристикой исключительно самих зарядов, и никак не зависит от поля.

Открытие данного факта позволило охарактеризовать электризацию тел и в дальнейшем было названо электрическим зарядом. Таким образом, по определению получается q1/q2 = F1/F2, где q1 и q2 — величина зарядов, помещаемых в одну точку поля, а F1 и F2 — силы, действующие на заряды со стороны поля.

Из подобных соображений были экспериментально установлены величины зарядов различных частиц. Условно положив в соотношение один из пробных зарядов равным единице, можно вычислить величину другого заряда, измерив соотношение F1/F2.

Через известный заряд можно охарактеризовать любое электрическое поле. Таким образом, сила, действующая на единичный пробный заряд, находящийся в состоянии покоя, называется напряженностью электрического поля и обозначается E. Из определения заряда получаем, что вектор напряженности имеет следующий вид: E = F/q.

Связь векторов j и E. Другая форма закона Ома

В однородном проводнике упорядоченное движение заряженных частиц будет происходить по направлению вектора E. А это значит, что векторы j и E будут сонаправлены. Как и при определении плотности тока, выделим в проводнике бесконечно малый цилиндрический объем. Тогда через поперечное сечение этого цилиндра будет проходить ток, равный j*dS, а напряжение, приложенное к цилиндру, будет равно E*dl. Также известна формула удельного сопротивления цилиндра.

Тогда, записав формулу силы тока двумя способами, получим: j = E/р, где величина 1/р носит название удельной электрической проводимости и является обратной к удельному электрическому сопротивлению. Ее принято обозначать σ (сигма) или λ (лямбда). Единицей измерения проводимости является См/м, где См — это Сименс. Единица, обратная Ом.

Таким образом, можно ответить на вопрос, поставленный выше, о законе Ома для неоднородной цепи. В таком случае на носителей тока будет действовать сила со стороны электростатического поля, которая характеризуется напряженностью E1, и другие силы, воздействующие на них со стороны другого источника тока, которые можно обозначить E2. Тогда Закон Ома применительно к неоднородному участку цепи будет иметь вид: j = λ(E1 + E2).

Подробнее о проводимости и сопротивлении

Способность проводника проводить электрический ток характеризуется его удельным сопротивлением, которое можно найти через формулу удельного сопротивления, или удельной проводимостью, рассчитывающейся как обратное проводимости. Величина данных параметров определяется как химическими свойствами материала проводника, так и внешними условиями. В частности температурой окружающей среды.

Для большинства металлов удельное сопротивление при нормальной температуре пропорционально ей, то есть р ~ T. Однако при низких температурах наблюдаются отклонения. У большого ряда металлов и сплавов при температурах, близких к 0°К, расчет сопротивления показывал нулевые значения. Это явление получило название сверхпроводимости. Таким свойством обладают, например, ртуть, олово, свинец, алюминий и др. Для каждого металла существует свое критическое значение температуры Tk, при которой наблюдается явление сверхпроводимости.

Также отметим, что определение удельного сопротивления цилиндра можно обобщить для проводов, состоящих из одного материала. В таком случае площадь поперечного сечения из формулы удельного сопротивления будет равна сечению провода, а l — его длине.

Зависимость сопротивления проводника от температуры

Каждое вещество имеет свое удельное сопротивление. Причем сопротивление будет зависеть от температуры проводника. Убедимся в этом, проведя следующий опыт.

Пропустим ток через стальную спираль. В цепи со спиралью подключим последовательно амперметр. Он покажет некоторое значение. Теперь будем нагревать спираль в пламени газовой горелки. Значение силы тока, которое покажет амперметр, уменьшится. То есть, сила тока будет зависеть от температуры проводника.

Изменение сопротивления в зависимости от температуры

Пусть при температуре 0 градусов, сопротивление проводника равняется R0, а при температуре t  сопротивление равно R, тогда относительное изменение сопротивления будет прямо пропорционально изменению температуры t:

(R-R0)/R=a*t.

В данной формуле а – коэффициент пропорциональности, который называют еще температурным коэффициентом.  Он характеризует зависимость сопротивления, которым обладает вещество, от температуры.

Температурный коэффициент сопротивления численно равен относительному изменению сопротивления проводника при нагревании его на 1 Кельвин.

Для всех металлов температурный коэффициент больше нуля. При изменениях температуры он будет незначительно меняться. Поэтому, если изменение температуры невелико, то температурный коэффициент можно считать постоянным, и равным среднему значению из этого интервала температур.

Растворы электролитов с ростом температуры сопротивление уменьшается. То есть для них температурный коэффициент будет меньше нуля.

Сопротивление проводника зависит от удельного сопротивления проводника и от размеров проводника. Так как размеры проводника при нагревании меняются незначительно, то основной составляющей изменения сопротивления проводника является удельное сопротивление.

Зависимость удельного сопротивления проводника от температуры

Попытаемся найти зависимость удельного сопротивления проводника от температуры.

Подставим в полученную выше формулу значения сопротивлений R=p*l/S R0=p0*l/S.

Получим следующую формулу:

p=p0(1+a*t).

Температурный коэффициент можно считать постоянным, следовательно, удельное сопротивление проводника будет прямо пропорционально температуре проводника.

Данная зависимость представлена на следующем рисунке.

Попробуем разобраться, почему увеличивается сопротивление

Когда мы повышаем температуру, то увеличивается амплитуда колебаний ионов в узлах кристаллической решетки. Следовательно, свободные электроны будут чаще с ними сталкиваться. При столкновении они будет терять направленность своего движения. Следовательно, сила тока будет уменьшаться.

Зависимость сопротивления проводника от температуры, широко используется в технике и физике. Например, в изготовлении термометров сопротивления.

Как найти внутреннее сопротивление формула в физике. Электродвижущая сила. Внутреннее сопротивление источника тока

Цель работы: изучить метод измерения ЭДС и внутреннего сопротивления источника тока с помощью амперметра и вольтметра.

Оборудование: металлический планшет, источник тока, амперметр, вольтметр, резистор, ключ, зажимы, соединительные провода.

Для измерения ЭДС и внутреннего сопротивления источника тока собирают электрическую цепь, схема которой показана на рисунке 1.

К источнику тока подключают амперметр, сопротивление и ключ, соединенные последовательно. Кроме того, непосредствен­но к выходным гнездам источника подключают еще и вольтметр.

ЭДС измеряют по показанию вольтметра при разомкнутом ключе. Этот прием определения ЭДС основан на следствии из за­кона Ома для полной цепи, согласно которому при бесконечно большом сопротивлении внешней цепи напряжение на зажимах источника равно его ЭДС. (См. параграф «Закон Ома для полной цепи» учебника «Физика 10»).

Для определения внутреннего сопротивления источника за­мыкают ключ К. При этом в цепи можно условно выделить два участка: внешний (тот, который подключен к источнику) и внутренний (тот, который находится внутри источника тока). Поскольку ЭДС источника равна сумме падения напряжений на внутрен­нем и внешнем участках цепи:

ε = U r +U R , то U r = ε -U R (1)

По закону Ома для участка цепи U r = I· r (2). Подставив равенство (2) в (1) получают:

I · r = ε U r , откуда r = (ε U R )/ J

Следовательно, чтобы узнать внутреннее сопротивление источника тока, необходимо пред­варительно определить его ЭДС, затем замкнуть ключ и измерить падение напряжения на внеш­нем сопротивлении, а также силу тока в нем.

Ход работы

1. Подготовьте таблицу для записи результатов измерений и вычислений:

ε

U r , B

i,a

r , Ом

    Начертите в тетради схему для измерения ЭДС и внутреннего сопротивления источника.

    После проверки схемы соберите электрическую цепь. Ключ разомкните.

    Измерьте величину ЭДС источника.

    Замкните ключ и определите показания амперметра и вольтметра.

    Вычислите внутреннее сопротивление источника.

  1. Определение эдс и внутреннего сопротивления источника тока графическим методом

Цель работы: изучить измерения ЭДС, внутреннего сопротивления и тока короткого замы­кания источника тока, основанный на анализе графика зависимости напряже­ния на выходе источника от силы тока в цепи.

Оборудование: гальванический элемент, амперметр, вольтметр, резистор

R 1 , переменный резистор, ключ, зажимы, металлический планшет, соединительные провода.

Из закона Ома для полной цепи следует, что напряжение на выходе источника тока зависит прямо пропорционально от силы тока в цепи:

так как I =E/(R+r), то IR + Ir = Е, но IR = U, откуда U + Ir = Е или U = Е – Ir (1).

Если построить график зависимости U от I, то по его точкам пересечения с осями координат можно определить Е, I К.З. — силу тока короткого замыкания (ток, который потечет в цепи источни­ка, когда внешнее сопротивление R станет равным нулю).

ЭДС определяют по точке пересечения графика с осью напряжений. Эта точка графика со­ответствует состоянию цепи, при котором ток в ней отсутствует и, следовательно, U = Е.

Силу тока короткого замыкания определяют по точке пересечения графика с осью токов. В этом случае внешнее сопротивление R = 0 и, следовательно, напряжение на выходе источника U = 0.

Внутреннее сопротивление источника находят по тангенсу угла наклона графика относи­тельно оси токов. (Сравните формулу (1) с математической функцией вида У = АХ +В и вспомни­те смысл коэффициента при X).

Ход работы

    Для записи результатов измерений подготовьте таблицу:

  1. После проверки схемы преподавателем соберите электрическую цепь. Ползунок переменного резистора установите в положение, при котором сопротивление цепи, подключенной к источ­нику тока, будет максимальным.
  2. Определите значение силы тока в цепи и напряжение на зажимах источника при максимальной величине сопротивления переменного резистора. Данные измерений занесите в таблицу.

    Повторите несколько раз измерения силы тока и напряжения, уменьшая всякий раз величину переменного сопротивления так, чтобы напряжение на зажимах источника уменьшалось на 0,1В. Измерения прекратите, когда сила тока в цепи достигнет значения в 1А.

    Нанесите полученные в эксперименте точки на график.

    Напряжение откладывайте по верти­кальной оси, а силу тока — по горизонтальной. Проведите по точкам прямую линию.

    Продолжите график до пересечения с осями координат и определите величины Е и, I К.З.

    Измерьте ЭДС источника, подключив вольтметр к его выводам при разомкнутой внешней це­пи. Сопоставьте значения ЭДС, полученные двумя способами, и укажите причину возможного расхождения результатов.

    Определите внутреннее сопротивление источника тока. Для этого вычислите тангенс угла на­клона построенного графика к оси токов. Так как тангенс угла в прямоугольном треугольнике равен отношению противолежащего катета к прилежащему, то практически это можно сделать, найдя отношение Е / I К.З

На концах проводника, а значит, и тока необходимо наличие сторонних сил неэлектрической природы, с помощью которых происходит разделение электрических зарядов .

Сторонними силами называются любые силы, действующие на электрически заряженные частицы в цепи, за исключением электростатических (т. е. кулоновских).

Сторонние силы приводят в движение заряженные частицы внут-ри всех источников тока: в генераторах, на электростанциях, в гальванических элементах, аккумуляторах и т. д.

При замыкании цепи создается электрическое поле во всех про-водниках цепи. Внутри источника тока заряды движутся под действием сторонних сил против кулоновских сил (электроны движут-ся от положительно заряженного электрода к отрицательному), а во всей остальной цепи их приводит а движение электрическое поле (см. рис. выше).

В источниках тока в процессе работы по разделению заряженных частиц происходит превращение разных видов энергии в электричес-кую. По типу преобразованной энергии различают следующие виды электродвижущей силы:

— электростатическая — в электрофорной машине, в которой происходит превращение механической энергии при трении в электрическую;

— термоэлектрическая — в термоэлементе — внутренняя энергия нагретого спая двух проволок, изготовленных из разных металлов, превращается в электрическую;

— фотоэлектрическая — в фотоэлементе. Здесь происходит превращение энергии света в элек-трическую: при освещении некоторых веществ, например, селена, оксида меди (I) , кремния наблюдается потеря отрицательного электрического заряда;

— химическая — в гальванических элементах, аккумуляторах и др. источниках, в которых происходит превращение химической энергии в электрическую.

Электродвижущая сила (ЭДС) — характеристика источников тока. Понятие ЭДС было введено Г. Омом в 1827 г. для цепей постоянного тока. В 1857 г. Кирхгофф определил ЭДС как работу сторонних сил при переносе единичного электрического заряда вдоль замкнутого контура:

ɛ = A ст /q ,

где ɛ — ЭДС источника тока, А ст — работа сторонних сил , q — количество перемещенного заряда.

Электродвижущую силу выражают в вольтах.

Можно говорить об электродвижущей силе на любом участке цепи. Это удельная работа сторонних сил (работа по перемещению единичного заряда) не во всем контуре, а только на данном участке.

Внутреннее сопротивление источника тока.

Пусть имеется простая замкнутая цепь, состоящая из источника тока (например, гальванического элемента, аккумулятора или генератора) и резистора с сопротивлением R . Ток в замкну-той цепи не прерывается нигде, следовательно, oн существует и внутри источника тока. Любой источник представляет собой некоторое сопротивление дли тока. Оно называется внутренним сопротивлением источника тока и обозначается буквой r .

В генераторе r — это сопротивление обмотки, в гальваническом элементе — сопротивление раствора электролита и электродов.

Таким образом, источник тока характеризуется величинами ЭДС и внутреннего сопротивлении, которые определяют его качество. Например, электростатические машины имеют очень большую ЭДС (до десятков тысяч вольт), но при этом их внутреннее сопротивление огромно (до со-тни Мом). Поэтому они непригодны для получения сильных токов. У гальванических элементов ЭДС всего лишь приблизительно 1 В, но зато и внутреннее сопротивление мало (приблизительно 1 Ом и меньше). Это позволяет с их помощью получать токи, измеряемые амперами.

ЭДС и напряжение. Внутреннее сопротивление источников питания.
Ликбез так ликбез!
Закон Ома. Вот я о чем.
О законе Ома мы уже говорили. Поговорим еще раз — с несколько иной стороны. Не вдаваясь в физические подробности и выражаясь простым кошачьим языком, закон Ома гласит: чем больше э.д.с. (электродвижущая сила), тем больше ток, чем больше сопротивление, тем меньше ток.
Переведя сие заклинание на язык сухих формул получаем:

I=E/R

где:I — сила тока,E — Э.Д.С. — электродвижущая силаR — сопротивление
Ток измеряется в амперах, э.д.с. — в вольтах, а сопротивление носит гордое имя товарища Ома.Э.д.с. — это есть характеристика идеального генератора, внутренне сопротивление которого принято считать бесконечно малым. В реальной жизни такое бывает редко, поэтому в силу вступает закон Ома для последовательной цепи (более знакомый нам):

I=U/R

где:U — напряжение источника непосредственно на его клеммах.
Рассмотрим простой пример.
Представим себе обычную батарейку в виде источника э.д.с. и включенного последовательно с ним некоего резистора, который будет олицетворять собой внутреннее сопротивление батарейки. Подключим параллельно батарейке вольтметр. Его входное сопротивление значительно больше внутреннего сопротивления батарейки, но не бесконечно большое — то есть, через него потечет ток. Величина напряжения, которую покажет вольтметр будет меньше величины э.д.с. как раз на величину падения напряжения на внутреннем воображаемом резисторе при данном токе.Но, тем не менее именно эта величина и принимается за напряжение батарейки.
Формула конечного напряжения при этом будет иметь следующий вид:

U(бат)=E-U(внутр)

Так как со временем у всех элементов питания внутреннее сопротивление увеличивается, то и падение напряжения на внутреннем сопротивлении тоже увеличивается. При этом напряжение на клеммах батарейки уменьшается. Мяу!
Разобрались!
Что же происходит, если вместо вольтметра к батарейке подключить амперметр? Так как собственное сопротивление амперметра стремится к нулю, мы фактически будем измерять ток, протекающий через внутреннее сопротивление батарейки. Так как внутренне сопротивление источника очень небольшое, измеренный при этом ток может достигать н ескольких ампер.
Однако следует заметить, что внутреннее сопротивление источника является таким же элементом цепи, как и все остальные. Поэтому при увеличении тока нагрузки падение напряжения на внутреннем сопротивлении также увеличится, что приводит к уменьшению напряжения на нагрузке. Или как мы, радиокоты, любим выражаться — к просадке напруги.
Чтобы изменение нагрузки как можно меньше влияло на выходное напряжение источника его внутреннее сопротивление стараются свести к минимуму.
Можно так подобрать элементы последовательной цепи, чтобы на каком-нибудь из них получить напряжение, уменьшенное, по сравнению с исходным, во сколько угодно раз.

Допустим, есть простейшая электрическая замкнутая цепь, включающая в себя источник тока, например генератор, гальванический элемент или аккумулятор, и резистор, обладающий сопротивлением R. Поскольку ток в цепи нигде не прерывается, то и внутри источника он течет.

В такой ситуации можно сказать, что любой источник обладает некоторым внутренним сопротивлением, препятствующим току. Это внутреннее сопротивление характеризует источник тока и обозначается буквой r. Для или аккумулятора внутреннее сопротивление — это сопротивление раствора электролита и электродов, для генератора — сопротивление обмоток статора и т. д.

Таким образом, источник тока характеризуется как величиной ЭДС, так и величиной собственного внутреннего сопротивления r – обе эти характеристики свидетельствуют о качестве источника.

Электростатические высоковольтные генераторы (как генератор Ван де Граафа или генератор Уимшурста), к примеру, отличаются огромной ЭДС измеряемой миллионами вольт, при этом их внутреннее сопротивление измеряется сотнями мегаом, потому они и непригодны для получения больших токов.

Гальванические элементы (такие как батарейка) — напротив — имеют ЭДС порядка 1 вольта, хотя внутреннее сопротивление у них порядка долей или максимум — десятка Ом, и от гальванических элементов поэтому можно получать токи в единицы и десятки ампер.

На данной схеме показан реальный источник с присоединенной нагрузкой. Здесь обозначены , его внутреннее сопротивление, а также сопротивление нагрузки. Согласно , ток в данной цепи будет равен:

Поскольку участок внешней цепи однороден, то из закона Ома можно найти напряжение на нагрузке:

Выразив из первого уравнения сопротивление нагрузки, и подставив его значение во второе уравнение, получим зависимость напряжения на нагрузке от тока в замкнутой цепи:

В замкнутом контуре ЭДС равна сумме падений напряжений на элементах внешней цепи и на внутреннем сопротивлении самого источника. Зависимость напряжения на нагрузке от тока нагрузки в идеальном случае линейна.

График это показывает, но экспериментальные данные на реальном резисторе (крестики возле графика) всегда отличаются от идеала:

Эксперименты и логика показывают, что при нулевом токе нагрузки напряжение на внешней цепи равно ЭДС источника, а при нулевом напряжении на нагрузке ток в цепи равен . Это свойство реальных цепей помогает экспериментально находить ЭДС и внутреннее сопротивление реальных источников.

Экспериментальное нахождение внутреннего сопротивления

Чтобы экспериментально определить данные характеристики, строят график зависимости напряжения на нагрузке от величины тока, затем экстраполируют его до пересечения с осями.

В точке пересечения графика с остью напряжения находится значение ЭДС источника, а в точке пересечения с осью тока находится величина тока короткого замыкания. В итоге внутреннее сопротивление находится по формуле:

Развиваемая источником полезная мощность выделяется на нагрузке. График зависимости этой мощности от сопротивления нагрузки приведен на рисунке. Эта кривая начинается от пересечения осей координат в нулевой точке, затем возрастает до максимального значения мощности, после чего спадает до нуля при сопротивлении нагрузки равном бесконечности.

Чтобы найти максимальное сопротивление нагрузки, при котором теоретически разовьется максимальная мощность при данном источнике, берется производная от формулы мощности по R и приравнивается к нулю. Максимальная мощность разовьется при сопротивлении внешней цепи, равном внутреннему сопротивлению источника:

Это положение о максимальной мощности при R = r, позволяет экспериментально найти внутреннее сопротивление источника, построив зависимость мощности, выделяемой на нагрузке, от величины сопротивления нагрузки. Найдя реальное, а не теоретическое, сопротивление нагрузки, обеспечивающее максимальную мощность, определяют реальное внутреннее сопротивление источника питания.

КПД источника тока показывает отношение максимальной выделяемой на нагрузке мощности к полной мощности, которую в данный момент развивает

Попробуем решить эту задачу на конкретном примере. Электродвижущая сила источника питания составляет 4,5 В. К нему подключили нагрузку, и через неё потёк ток, равный 0,26 А. Напряжение при этом стало равным 3,7 В. Первым делом, представим себе, что внутри источника спрятана последовательная цепь из идеального источника напряжения в 4,5 В, внутреннее сопротивление которого равно нулю, а также резистора, номинал которого и требуется найти. Понятно, что на самом деле это не так, но для расчётов аналогия вполне сойдёт.

2 шаг

Запомните, что буквой U обозначают только напряжение под нагрузкой. Для обозначения же электродвижущей силы зарезервирована другая буква – E. Абсолютно точно её измерить невозможно, потому что потребуется вольтметр с бесконечным входным сопротивлением. Даже у электростатического вольтметра (электрометра) оно огромно, но не бесконечно. Но одно дело – абсолютно точно, а другое – с точностью, приемлемой на практике. Второе вполне осуществимо: нужно лишь, чтобы внутреннее сопротивление источника было пренебрежимо мало по сравнению с внутренним сопротивлением вольтметра. А пока суть да дело, посчитаем разницу между ЭДС источника и его напряжением под нагрузкой, потребляющей ток в 260 мА. E-U = 4,5-3,7 = 0,8. Это и будет падение напряжения на том “виртуальном резисторе”.

3 шаг

Ну а дальше всё просто, ибо в дело вступает классический закон Ома. Помним, что ток через нагрузку и “виртуальный резистор” одинаков, ведь они соединены последовательно. Падение напряжения на последнем (0,8 В) делим на силу тока (0,26 А) и получаем 3,08 Ома. Вот и готов ответ! Можно ещё посчитать, какая мощность рассеивается на нагрузке, а какая – бесполезно на источнике. На нагрузке рассеивается: 3,7*0,26=0,962 Вт. На источнике: 0,8*0,26=0,208 Вт. Процентное соотношение между ними вычислите самостоятельно. Но эта не единственный вид задач на нахождение внутреннего сопротивления источника. Есть и такие, в которых вместо силы тока указано сопротивление нагрузки, а остальные исходные данные такие же. Тогда надо вначале проделать ещё одно вычисление. Приведённое в условии напряжение под нагрузкой (не ЭДС!) поделить на сопротивление нагрузки. И получится сила тока в цепи. После чего, как говорят физики, “задача сведена к предыдущей”! Попробуйте составить такую задачу и решить её.

Расчет сопротивления — AP Physics C Electricity

Если вы считаете, что контент, доступный через Веб-сайт (как определено в наших Условиях обслуживания), нарушает или другие ваши авторские права, сообщите нам, отправив письменное уведомление («Уведомление о нарушении»), содержащее в информацию, описанную ниже, назначенному ниже агенту. Если репетиторы университета предпримут действия в ответ на ан Уведомление о нарушении, оно предпримет добросовестную попытку связаться со стороной, которая предоставила такой контент средствами самого последнего адреса электронной почты, если таковой имеется, предоставленного такой стороной Varsity Tutors.

Ваше Уведомление о нарушении прав может быть отправлено стороне, предоставившей доступ к контенту, или третьим лицам, таким как в качестве ChillingEffects.org.

Обратите внимание, что вы будете нести ответственность за ущерб (включая расходы и гонорары адвокатам), если вы существенно искажать информацию о том, что продукт или действие нарушает ваши авторские права. Таким образом, если вы не уверены, что контент находится на Веб-сайте или по ссылке с него нарушает ваши авторские права, вам следует сначала обратиться к юристу.

Чтобы отправить уведомление, выполните следующие действия:

Вы должны включить следующее:

Физическая или электронная подпись правообладателя или лица, уполномоченного действовать от их имени; Идентификация авторских прав, которые, как утверждается, были нарушены; Описание характера и точного местонахождения контента, который, по вашему мнению, нарушает ваши авторские права, в \ достаточно подробностей, чтобы позволить репетиторам университетских школ найти и точно идентифицировать этот контент; например нам требуется а ссылка на конкретный вопрос (а не только на название вопроса), который содержит содержание и описание к какой конкретной части вопроса — изображению, ссылке, тексту и т. д. — относится ваша жалоба; Ваше имя, адрес, номер телефона и адрес электронной почты; а также Ваше заявление: (а) вы добросовестно считаете, что использование контента, который, по вашему мнению, нарушает ваши авторские права не разрешены законом, владельцем авторских прав или его агентом; (б) что все информация, содержащаяся в вашем Уведомлении о нарушении, является точной, и (c) под страхом наказания за лжесвидетельство, что вы либо владелец авторских прав, либо лицо, уполномоченное действовать от их имени.

Отправьте жалобу нашему уполномоченному агенту по адресу:

Чарльз Кон Varsity Tutors LLC
101 S. Hanley Rd, Suite 300
St. Louis, MO 63105

Или заполните форму ниже:

Q = It E = расчеты QV Закон Ома V = IR исследующие факторы, влияющие на сопротивление Графические характеристики ВАХ диод омический проводник лампа накаливания igcse / gcse 9-1 Physics revision notes

Электричество 3: Закон Ома, экспериментальные исследования сопротивления

а также I-V графики и расчеты с использованием I = V / R, Q = It и E = QV

Редакция Доктора Брауна по физике Банкноты

Подходит для курсов GCSE / IGCSE Physics / Science или их эквивалент

Что такое закон Ома? Как вы делаете расчеты по закону Ома / с?

Какие факторы влияют на стойкость схема?

Как построить и использовать схему для исследовать закон Ома?

Как рассчитать количество эл. заряд движется по цепи?


Субиндекс этой страницы

1.Закон Ома, простая схема исследования и расчеты V = IR

2. Движение и единица заряда, кулон, расчеты с использованием Q = It

3. Возможная разница и передача электроэнергии, E = QV расчеты

4а. Электрическое сопротивление — задействованных факторов

4б. Изучение сопротивление провода при постоянной температуре, различной длины и ширины

4с.Изучение вольт-амперные характеристики провода — объяснение графика

4д. Расследование ВАХ металлической лампы накаливания — график

4e. Изучение вольт-амперные характеристики диода — объяснение графика

См. Также ПРИЛОЖЕНИЕ 1 для обзора всей электроэнергии уравнения вам могут понадобиться


1а.Ома Закон (и упоминание других единиц, рассматриваемых в других разделах)

Закон Ома гласит, что ток через провод между двумя точками прямо пропорционален напряжению на две точки в цепи.

Он включает в себя самое фундаментальное уравнение что нужно знать для расчета электричества.

Математически это можно выразить как: я = V / R

перестановки: В = ИК а также R = V / I

I = ток в амперах, амперах, А ; мера скорости потока электрического заряда.

В = разность потенциалов, п.о., вольт, В ; мера потенциальной энергии, передаваемой электрическому заряду течет.

Разность потенциалов в цепи это энергия , передаваемая на кулон электрического заряда , что течет между двумя точками в электрической цепи.

Кулон ( C ) — это единица электрического заряда (см. Q = Примечания к уравнению).

Передаваемая энергия рассчитывается от п.о. и количество электрического заряда ( Q ) перемещен p.d. в В (см. E = QV примечания к уравнениям).

R = сопротивление провода, Ом, Ом ; мера сопротивления проводника препятствовать поток заряда.

Чем больше сопротивление резистор, тем больше он сопротивляется и замедляет ток электричества.

Закон Ома означает, что R в этом уравнении является константой, не зависящей от величины протекающего электрического тока.

Закон правильно применяется к так называемому омическому сопротивлению . проводники , где протекающий ток прямо пропорционален приложенная разность потенциалов, но некоторые резисторы не подчиняются этому закону, например нагретая нить лампочка.

1b.Просто эксперимент по измерению сопротивления отдельного компонента

Если вы настроили контур 31 (правая диаграмма), вы можете измерить сопротивление постоянного резистора [R].

Изменяя напряжение источника питания с помощью переменной резистора, вы легко можете получить множество пар показаний p.d. (V) и текущее (А).

Затем используйте уравнение закона Ома (R = V / I), чтобы вычислить значение фиксированное сопротивление.

Затем вы можете усреднить значения R, рассчитанные для более надежный результат.

Более подробная информация приведена ниже, чтобы получить полную ВАХ графики, а также как получить сопротивление графическим методом.

Это основная установка для исследования вольт-амперные характеристики любого компонента Р.

1c. Примеры расчеты с использованием Ома Закон V = IR

Q1 Когда стр.d. от 4,5 В приложен к сопротивлению, течет ток 0,5 А.

Какое значение имеет резистор?

R = V / I = 4,5 / 0,5 = 9,0 Ом

Q2 Сопротивление имеет значение 50 Ом.

Какой п.д. должен быть применен к нему чтобы через него протекал ток 5,0 А?

В = ИК = 5 x 50 = 250 V

3 квартал А п.d. 240 В подается на резистор нагревательного элемента сопротивлением 30 Ом.

Сколько тока проходит через обогреватель?

I = V / R = 240/30 = 8.0 А

4 квартал Три батареи на 1,5 В были подключены последовательно к трем лампочкам.

Если амперметр измеряет ток 0,50 А, какое сопротивление у каждой лампочки?

I = V / R, поэтому R = V / I = (3 x 1.5) / 0,50 = 9,0 Ом

Так как общее сопротивление = сумма сопротивления, сопротивление каждой лампочки = 9,0 / 3 = 3.0 Ом

Q5


ВЕРХ СТРАНИЦЫ и субиндекс


2. Движение заряда

2а. Расчет заряда, проходящего через точку в цепи Q = It

Ток (I в амперах) — это скорость протекания электрического заряжать вокруг цепи.

Чем больше поток заряда в данный момент времени, тем больше электрический ток.

Скорость протекания электрического заряда измеряется в кулонах в секунду.

Вы можете рассчитать заряд, проходящий через точку в цепи в заданное время по формуле

Q = Это

, где Q = электрический заряд в кулонах ( C ) — блок электрического заряда

I = ток в амперах ( A, ) и t = время ( с )

перестановки из Q = It, I = Q / т и t = Q / I

Ток в 1 А равен скорость потока заряда 1 Кл / с.

2b. Примеры расчетных вопросов с участием уравнение Q = It

Q1 Если ток 3,0 А проходит через прибор в течение 1 часа 30 минут, сколько электрического заряда передается в процессе?

Q = It, Q = 3,0 x 1,5 x 60 x 60 = 16 200 C = 1,62 х 10 4 С

Q2 Если Заряд 9000 C проходит через точку в электрической цепи в 12.0 минут, какой текущий поток?

I = Q / t = 9000 / (12 x 60) = 9000/720 = 12,5 А

Q3 Сколько времени займет электрическая цепь в минутах и ​​секундах ток 20.0 А передать 5000 С заряда?

t = Q / I = 5000/20 = 250 секунд = 4 минут и 10 секунд

Q4 Портативный компьютер зарядное устройство пропускает ток 1.20 А на 30 минут с выходом p.d. 15.0 В.

(a) Подсчитайте, сколько заряда перенесена в компьютерную батарею.

Q = It = 1,2 х 30 х 60 = 2160 С

(b) Какое сопротивление зарядное устройство?

В = ИК, R = V / I = 15 / 1,2 знак равно 12,5 Ом

(c) Когда аккумулятор ноутбука полностью заряжено в нем хранит 3000 с.

Сколько времени потребуется, чтобы полностью заряжать разряженный аккумулятор?

Q = It, t = Q / I = 3000 / 1,2 = 2500 с (41 мин 40 с)

Q5


ВЕРХ СТРАНИЦЫ и субиндекс


3. Возможная разница и передача энергии

3а.Введение в электрическую энергию перевод

Передача энергии на единицу заряда = разность потенциалов (п.о.) и расчеты на основе E = QV

В предыдущем разделе мы рассмотрели, как рассчитать количество заряда, движущегося в цепи, но ничего не сказал об энергии переведен.

Напоминания :

Электрические цепи, используемые термины, условные обозначения цепей, параллельные объяснение цепей и последовательных цепей

Разница потенциалов (стр.d. в вольт, В ) — энергия, передаваемая на единицу заряда в виде электрического заряд перемещается из одной точки в другую в электрической цепи.

Измеряется вольтметром, который всегда подключается параллельно компоненту схемы.

Передача электрического тока энергия

Подумайте обо всех электрических бытовые приборы, которыми вы пользуетесь — все они нуждаются в подаче энергии для работы!

Блок питания работает на заряжается и передает ему энергию.

Работа должна производиться на зарядке для увеличения его потенциальной энергии.

Электрический заряд измеряется в кулоны (C)

Заряд и его движение уже было рассмотрено в раздел 2 (Q = It).

Заряды передают энергию компонентов по мере их прохождения, выполняя работу против сопротивления компонент.

Если работа сделана, то энергия переведен.

Если электрический заряд испытывает разность потенциалов, этот заряд будет течь, передавая энергию.

Энергия подается из энергоаккумулятора источник питания — аккумулятор, электросеть и т. Д.

При прохождении заряда по любой п.d. Падение высвобождает энергию (с более высокого уровня потенциальной энергии на более низкий).

например в тонкой проволоке сопротивление , выделяется тепло.

Разница потенциалов между двумя баллов равняется проделанной работе на единицу заряда.

разность потенциалов (В) = проделанная работа ( энергия передается в Дж) заряд (C)

т.е. 1 вольт соответствует 1 джоуля за кулон или V = J / C

Чем больше падение п.д., тем больше энергия передан, потому что заряд начинается с большим потенциалом энергия.

Следовательно, блок питания с большим источником шт. (В) может подавать на больше энергии в схему на единицу электрического заряда ( кулон, В).

Чем больше p.d., тем больше энергии такое же количество электрический заряд может нести.

3b.Еще одно уравнение для расчета электрического передача энергии

В количество переносимой энергии можно рассчитать по формуле:

переданная энергия = заряд x потенциал разница.

E = QV (поэтому Q = E / V и V = E / Q )

E = передаваемая энергия в джоулях ( Дж, )

Q = количество электрический заряд в кулонах ( C )

В = разность потенциалов ( В )

Отметив, что: V = E / Q = энергия, передаваемая на единицу заряда (J / C)

Попутно и несколько напоминаний:

Чем больше энергии передается в данном время, тем больше мощность устройства или электрического прибора.

г. V говорит вам, сколько энергия, передаваемая на каждую единицу электрического заряда,

так, В = E / Q , (ед. J / C), см. Расчеты E = QV ниже).

Текущий I говорит вам, сколько заряд проходит заданную точку в цепи за единицу времени (кулонов в секунду, К / с ).

Это означает, что оба p.d. В и текущий I влияет на скорость передачи энергии прибору из накопителя электроэнергии в другие накопители энергии.

И несколько математических связей на основании раздела 2. Q = It, а здесь, в разделе 3, E = QV

Из Q = It и E = QV, подставив дает E = ItV,

так (i) E = IVt (I в A, t в с, В в вольтах)

Перестановка E = IVt дает IV = Е / т

Это соединяется с уравнения для мощности

(ii) Мощность = энергия переведено / время получено = E / t (Дж / с), и

( iii ) Мощность = ток x напряжение = P (Вт) = I (A) x V (В), P = IV

Из (ii) и (iii) E / t = IV, поэтому E = IVt , что является уравнением (i) !!!

3c.Расчет q на основе E = QV (иногда с участием других уравнений электричества)

Q1 Электродвигатель Модель автомобиля питается от аккумулятора 1,5 В.

Если через цепь двигателя в движущемся вагоне,

(а) сколько энергии передается?

E = QV = 120 x 1,5 = 180 Дж

(b) Опишите вероятный накопитель энергии меняется, когда машина работает.

Химическая потенциальная энергия запас батареи уменьшается и превращается в электрическую энергию.

Накопитель кинетической энергии автомобиль увеличивается из-за потери тепла от трения и звуковой энергии переданы в накопитель тепловой энергии окрестностей.

Q2 Какое количество заряд необходим для передачи 500 Дж энергии, если п.о.цепи 24,0 В?

E = QV, Q = E / V = 500/24 ​​= 20,8 С (3 SF)

Q3 Какой потенциал разность требуется в цепи для передачи 2000 Дж энергии с заряд 50 кулонов?

E = QV, V = E / Q = 2000/50 = 40 В

4 квартал А 12.Батарея 0 В пропускает через лампу ток 2,0 А в течение 5 минут.

(а) Рассчитайте, сколько заряда прошло через лампу.

Q = It = 2 x 5 x 60 = 600 С

(b) Рассчитайте, сколько электроэнергии был перенесен лампой.

Два пути:

(i) E = QV = 600 x 12 = 7200 Дж, самый простой, но вы можете рассчитать его, не зная Q:

(ii) E = IVt = 2 x 12 x 5 x 60 = 7200 Дж

Q5 Устройство имеет мощность 1.5 кВт и работает от сети 230 В.

Если прибор используется в течение 15 минут, сколько заряда прошло по цепи?

1,5 кВт ≡ 1500 Вт ≡ 1500 Дж / с

Общая переданная энергия = мощность x время = 1500 x 15 x 60 = 1 350 000 Дж

E = QV, поэтому Q = E / V = ​​1350 000/230 = 5870 С (3 SF)

Ответ можно рассчитать по другой маршрут

P = IV, I = P / V = ​​1500/230 = 6.522 А

Q = It = 6,522 x 15 x 60 = 5870 С (3 SF)

Q6

3d. Еще немного о разности потенциалов — действие двух последовательно соединенных резисторов

Схема 41 показывает два подключенных резистора последовательно.

Справа показано, что происходит с p.d. по часовой стрелке по цепи (направление условного тока).

Повышается потенциальный запас аккумулятора разность зарядных потенциалов заряда до 12 В.

По мере прохождения заряда через 1-й резистор R 1 , он теряет энергию и п.д. падает на 8 В до п.п. из 4 В.

По мере прохождения заряда через 2-ю резистор R 1 , он снова теряет энергию и п.о. падает на 4 В до p.d. из 0 В.

Пока есть полная цепь, процесс повторяется.

Поскольку E = QV, выделяется вдвое больше энергии резистором R 1 (p.d. 8 В), чем R 2 (p.d. 4 В) для тот же ток.


ВЕРХ СТРАНИЦЫ и субиндекс


4. Электрическое сопротивление — эксперименты по исследовать ВАХ различных сопротивлений и достоверность, или в противном случае Закона Ома

4а. Что влияет на сопротивление провода? Сопротивление постоянный?

и s — графики вольт-амперных характеристик (ВАХ) объяснил

Сопротивление цепи зависит от несколько факторов:

(i) толщина сопротивления провод — для однокомпонентного резистора

(ii) длина сопротивления провод — для однокомпонентного резистора

(iii) если более одного сопротивления, они подключены последовательно или параллельно?

(iv) температура компонента действует как сопротивление

Раздел 4.описывает и объясняет несколько примеров графиков ВАХ — которые можно исследовать с помощью схемы 31 (справа)

Схема 31 справа показывает как можно исследовать изменение тока через сопротивление (или любой компонент) при изменении разности потенциалов.

Графики разности потенциалов тока используются, чтобы показать, как ток через компонент изменяется в зависимости от разности потенциалов на нем.

Сопротивление некоторых резисторов / компонентов действительно меняется. как действующий и п.д. изменения например диодная или лампа накаливания.

Узнайте, как и почему в разделах 4d. и 4e.


ВЕРХ СТРАНИЦЫ и субиндекс


4b. Исследование электрического сопротивления провода — изменение длины или ширины

Схема 30 показывает, как исследовать сопротивление провода

Относительно тонкая проволока закреплена на каждый конец на линейке с разметкой в ​​миллиметрах с помощью зажимов типа «крокодил».

Вам понадобится амперметр для измерения ток в амперах и вольтметр для измерения p.d. через провод в вольт.

Провод подключается в серии с аккумуляторным блоком питания, переключателем и амперметром для измерения силы тока течет по проводу в амперах.

Вольтметр , для измерения p.d, подключены по параллельно через резистивный провод.

Обратите внимание, что амперметр всегда подключается к последовательно с компонентом , но вольтметр всегда подключается параллельно по любому исследуемому компоненту.

Один конец провода, подключенный через вольтметр закреплен (слева), но на другом конце есть зажим типа «крокодил», который действует как подвижный точку контакта для размещения на различном расстоянии вдоль провода сопротивления от слева направо.

Замкните выключатель, чтобы замкнуть цепь и начинаем снимать показания.

Лучше всего открыть переключатель между показания, чтобы свести к минимуму риск нагрева провода.

Вы изменяете расстояние d (мм) от слева (0 мм) в точку дальше вправо и возьмите серию пар из п.d и текущие показания, например каждые 50 мм (можно работать в см, получается нет разницы!).

Используя закон Ома, вы вычисляете сопротивление в омах из уравнения R = V / I

Ты затем можно построить график зависимости сопротивления (Ом) от длины провода d (мм) — показано справа.

Вы должны обнаружить, что график является линейным с его начало координат x, y в точке 0,0.

Это означает, что сопротивление пропорционально длине провода .

Если не закрепить провод точно на 0 мм, график по-прежнему должен быть линейным, но начало линии не будет быть 0,0.

Если повторить эксперимент с провода разного диаметра, вы должны обнаружить, что градиент становится ниже, толще проволока.

При той же длине провода сопротивление меньше толще проволока — хорошая аналогия — легкость, с которой вода течет через труба тонкого или большего диаметра.


ВЕРХ СТРАНИЦЫ и субиндекс


4c. Изучение ток — напряжение характеристики проволоки

Это эксперимент по закону Ома

Схема 31 показывает, как исследовать зависимость I от V для сопротивления

.

Расследование сводится к поиску из …

… как ток течет через резистор меняются в зависимости от разности потенциалов на нем?

Постоянный резистор представляет собой ‘компонент’ в цепи и должен иметь постоянную температуру на протяжении всего эксперимента (см. ниже температура эффекты).

В этом случае простой проволочный резистор соединены последовательно с блоком питания и амперметром.

г. измеряется по фиксированному сопротивление с вольтметром,

Тем не мение, также подключенный последовательно, добавлен переменный резистор, так что вы можете удобно изменить разность потенциалов и тем самым изменить ток протекает через компонент.

Это позволяет собрать целую серию пар показаний I и V, с помощью которых можно построить подходящие графики — в этом случай V против I, но часто делается как I против V.

Используя переменный резистор, вы постепенно увеличивайте разность потенциалов на компоненте, принимая соответствие текущему чтению, например увеличиваясь на 0,5 В. за раз. Повторите каждый читать дважды и использовать среднее значение.

После этого можно поменять местами клеммы аккумулятора. и повторить все показания.

Если вы построите p.d. по сравнению с текущим, график линейный , если он подчиняется закону Ома — тогда он называется ‘ омический проводник ‘.

Это я изобразил графиком вверху справа, а градиент равен сопротивлению в Ом .

Это соответствует закону Ома уравнение V = IR , поэтому градиент равен R.

Если график остается линейным, сопротивление остается постоянным.

Этот график не представляет показания сняты после перестановки полюсов аккумулятора.

Однако показывает, как получить значение сопротивления графическим методом.

Это линейный график и фраза линейный компонент может использоваться.

Для таких компонентов, как провод, который не нагревается, вы должны получить линейный график p.d. (V) против I (A) с градиент R (Ω). (правый график).

Вы должны убедиться, что провод не сильно нагревается — если начинает нагреваться, сразу отключите резистор («выключите») и дайте ему остыть.

Если вы построите график зависимости I от V, градиент будет 1 / R (обратная величина сопротивление), линейный график .

Этот график показывает, что вы получаете построение всех данных, включая показания I-V, снятые после реверсирования клеммы аккумулятора.

График (1) является построенный на поперечной оси. Верхняя правая половина — это ваш первый набор результатов, затем вы переверните клеммы на источнике питания и повторите эксперимент дает нижнюю левую часть графика.

Обратите внимание, что вы получите только линейный график, если температура проволоки остается постоянной .

Когда ток (А) пропорционален к p.d (V) он описывается как омический провод (подчиняется закону Ома!).

С помощью схемы 31 вы можете проверить любой резистор или любой другой тип компонента схемы, и результаты обсуждается ниже, начиная с резюме факторов, влияющих на сопротивление.

Так, сопротивление омического проводника, например цепь компонент не меняется независимо от того, проходит ли ток — постоянный градиент 1 / R для графика 1.

Это ожидаемые линейные графики для фиксированный резистор с использованием схемы 31 выше.

Думая против часовой стрелки на диаграмме, разные линии графика могут отображать убывающее сопротивление е.грамм. (i) резистивный провод становится короче при том же диаметре, или (ii) увеличение диаметра при фиксированной длине провода.

При постоянной температуре ток, протекающий через омический проводник прямо пропорционален разности потенциалов на это — уравнение: В = IR или I = V / R .

Однако это только правда, что дает линейный график, если температура не изменение.

Комментарии к переменным в этом частный эксперимент по закону Ома

Ток всегда определяется комбинация п.о. (В) и сопротивление R (Ом).

Независимая переменная что мы изменяем или контролируем в эксперименте — в этом случае вы можете Считайте это р.д. управляется переменным резистором.

Одно из соглашений состоит в том, чтобы построить независимая переменная на оси абсцисс, а зависимая переменная на оси ось y.

Этот означает, что сопротивление R, является обратной величиной градиента — немного больше неудобно рассчитать сопротивление, чем на графике V в зависимости от I, где градиент — это сопротивление. Закон Ома: I = В / р.

Зависимая переменная — это то, что мы тестируем или измеряем в эксперименте, это ток I (A), который зависит от настройки переменного резистора, который, в свою очередь, контролирует разность потенциалов на резисторе.

Контрольные переменные — это то, что мы сохраняем то же самое во время эксперимента, чтобы убедиться, что это честный тест например в этом случае сохраняется температура провода и постоянная, НЕ должна изменяться — не меняйте провод и не допускайте нагреть.


ВЕРХ СТРАНИЦЫ и субиндекс


4г. Исследование вольт-амперных характеристик металла лампа накаливания

Когда электрический заряд проходит через высокое сопротивление, как тонкая металлическая нить лампы накаливания, она передает часть электроэнергия к накопителю тепловой энергии нити накала. Электрический заряд работают против сопротивления .

Схема 45 показывает, как можно исследовать характеристики разности токов и потенциалов нить лампочка.

Вольтметр подключен параллельно термистор, п.д. В измеряется в вольтах ( В ).

Переменный резистор позволяет варьировать п.д. и текущий поток.

Амперметр, подключенный последовательно, дает вам текущее значение I в амперах ( A, ).

В прохождение тока нагревает нить накала и поднимается в температура вызывает повышение сопротивления . Итак, нить Лампа — это неомический провод .

Этот «эффект нагрева» влияет на все резисторы.

По мере увеличения тока выделяется больше тепловой энергии, и нить накаливания становится все горячее и горячее, поэтому дальнейшее повышение температуры еще больше увеличивает сопротивление.

Это уменьшает скорость, с которой ток увеличивается с увеличение разности потенциалов .

Следовательно, градиент I-V кривая графика уменьшается, и все больше с увеличением температура — график 2. Это нелинейный график .

Если градиент меняется, тогда сопротивление меняется.

График (2) является построенный на поперечной оси.Верхняя правая половина — это ваш первый набор результатов, затем вы переверните клеммы на источнике питания и повторите эксперимент дает нижнюю левую часть графика.

Фраза нелинейная компонент может быть использован.

Когда ток (А) НЕ пропорциональная p.d (V) лампа накаливания описывается как безомический провод (не подчиняется закону Ома!).

У вас такой же график в форме треугольника. для термистор.

Теория — со ссылкой на схему металлической конструкции

Металлическая кристаллическая решетка состоит из неподвижных ионов и свободно движущихся электроны между ними. С повышением температуры ионы металла колеблются сильнее. сильно, в котором электроны сталкиваются, и это препятствует прохождению электронов, уменьшая поток плата. По мере увеличения тока вибрации увеличиваются, вызывая больше электрической энергии, которая будет преобразована в тепло — увеличивая температура И сопротивление металлической нити, тем самым еще больше понижая ток.

Итак, увеличение i n по температуре увеличивает сопротивление лампы накаливания (или большинства других резисторы) и снижает ток, протекающий для заданного p.d.

Если резистор становится слишком горячим, ток почти не будет течь.

Есть одно важное исключение к этому «правилу», см. примечания к термистор, где сопротивление фактически падает с повышение температуры.

Лампа накаливания — лишь одна из многих примеры были энергия передается с пользой , НО есть всегда теряется тепловая энергия в накопитель тепловой энергии устройства и Окрестности .

Нить накала часто изготавливается из металлический вольфрам, плавящийся при> 3400 o C и ярко светящийся при 2500 o C, но он все равно испаряется очень медленно. Инертный газ например, добавляется аргон или азот, чтобы уменьшить это испарение — любое испаренные атомы вольфрама попадают в инертный (и, следовательно, неокисляющий) Ar или N 2 молекул и, надо надеяться, снова конденсируются на нити.

См. Сохранение энергии, передача-преобразование энергии, эффективность — расчеты


ВЕРХ СТРАНИЦЫ и субиндекс


4e. Исследование вольт-амперных характеристик диода

Модель ток через диод протекает только в одном направлении — см. график 3.

Сопротивление в обратном направлении очень высока — следовательно, это фактически «односторонняя» система.

Схема 43 показывает, как можно исследовать вольт-амперная характеристика диода.

Вольтметр подключен параллельно термистор, п.д. В измеряется в вольтах ( В ).

Переменный резистор позволяет варьировать п.д. и текущий поток.

Амперметр, подключенный последовательно, дает вам текущее значение I в амперах ( A, ).

Диод имеет очень высокое сопротивление в обратное направление .

Также есть порог р.о. (например, 1,4 В) до того, как ток вообще потечет — внимательно посмотрите на график — есть короткий горизонтальный участок, прежде чем ток поднимется с нуля и со временем становится линейным.

Таким образом, вы получаете верхнюю правую часть графика 3 по сравнению с графиками 1 и 2 выше.

Это потому, что когда вы делаете экспериментируйте, используя схему, описанную выше, по изменению подключений, вы не обнаружите никаких текущих потоков при изменении p.d.

Его нелинейный График .

Если градиент меняется, то сопротивление меняется.

Когда ток (А) НЕ пропорциональный p.d (V), диод описывается как безомический провод (не подчиняется закону Ома!).

Фраза Нелинейный компонент может быть использовано.

График (3) является построенный на поперечной оси. Верхняя правая половина — это ваш первый набор результатов, затем вы переверните клеммы на источнике питания и повторите эксперимент дает нижнюю левую часть графика.

Так как ток течет только в одном направлении через диод его можно использовать для преобразования переменного тока в постоянный Текущий.


ВЕРХ СТРАНИЦЫ и субиндекс


Практическая работа по помощь в развитии ваших навыков и понимания, возможно, включала следующее:

с использованием ламп накаливания и резисторов для исследования разности потенциалов — Текущий характеристики,


ПРИЛОЖЕНИЕ 1: Важные определения, описания, формулы и ед.

Примечание: Вы можете / можете нет (но не волнуйтесь!), столкнулись со всеми этими терминами, это зависит от как далеко продвинулась ваша учеба.В вашем курсе вам может не понадобиться каждая формула — решать вам.

V разность потенциалов ( p.d ., обычно называемая « напряжение ») — это движущий потенциал, который перемещает электрический заряд вокруг цепь — обычно электронов .

Возможная разница — это работа, выполненная в перемещение единицы заряда.

Показывает, сколько энергии передается за единицу заряда, когда заряд перемещается между двумя точками в цепи е.грамм. между выводами аккумуляторной батареи.

г. в любой части цепи измеряется в вольтах, В .

Я ток — это скорость протекания электрического заряда в кулонов в секунду ( C / s ), измеряется в амперах (амперах, A, ).

Количество переданного электрического заряда a give time = текущий расход в амперах x прошедшее время в секундах

Формула соединения: Q = Это , I = Q / t, t = Q / I, Q = перемещаемый электрический заряд кулонов ( C ), время т ( с )

R сопротивление в цепи, измеренное в Ом ( Ом ).

Сопротивление замедляет прохождение электрического заряда — он противостоит потоку электрического заряда .

Формула соединения: В = ИК , I = V / R, R = V / I (Это формула для Закон Ома)

P является мощность , передаваемая цепью = показатель энергии передача ( Дж / с, ) и измеряется в Вт, ( Вт, ).

Формула соединения: P = IV , I = P / V, V = P / I также P = I 2 R (см. также P = E / t ниже)

E = QV , энергия, передаваемая количеством электрического заряда потенциалом разность вольт.

переданная энергия (джоулей) = количество электрического заряда (кулоны) x разность потенциалов (вольт)

Q = E / V, V = E / Q, E = передача энергии в джоулях ( Дж, ), Q = перемещенный электрический заряд ( C ), В = p.d. ( В )

E = Pt , P = E / t, t = E / P, где P = мощность ( Вт, ), E = переданная энергия ( Дж) , t = затраченное время ( с )

Передаваемая энергия в джоулях = мощность в ваттах. x время в секундах

Формула связи: Поскольку E = Pt и P = IV, переданная энергия E = IVt


ВЕРХ СТРАНИЦЫ и субиндекс


Что дальше?

Электричество и ревизия магнетизма индекс нот

1.Полезность электроэнергии, безопасность, передача энергии, расчеты стоимости и мощности, P = IV = I 2 R, E = Pt, E = IVt

2. Электрические схемы и как их рисовать, условные обозначения схем, параллельность схемы, объяснение последовательных схем

3. Закон Ома, экспериментальные исследования сопротивление, I-V графики, расчеты V = IR, Q = It, E = QV

4. Схемы устройств и как они используются? (е.грамм. термистор и LDR), соответствующие графики gcse Physical Revision

5. Подробнее о последовательных и параллельных цепях. электрические схемы, измерения и расчеты gcse физика

6. Электроснабжение «Национальной сети», экология вопросы, использование трансформаторов gcse примечания к редакции физики

7. Сравнение способов получения электроэнергии gcse Заметки о пересмотре физики (энергия 6)

8.Статическое электричество и электрические поля, использование и опасность статического электричества gcse примечания к редакции физики

9. Магнетизм — магнитные материалы — временные (индуцированные) и постоянные магниты — использует gcse физика

10. Электромагнетизм, соленоидные катушки, применение электромагнитов gcse примечания к редакции физики

11. Моторное воздействие электрического тока, электродвигатель, громкоговоритель, правило левой руки Флеминга, F = BIL

12.Эффект генератора, приложения, например. генераторы генерация электричества и микрофон gcse физика

ВСЕ мои GCSE Примечания к редакции физики

ИЛИ воспользуйтесь [GOGGLE ПОИСК]



Версия IGCSE заметки по закону Ома исследования сопротивления V = IR Q = It вычисления KS4 физика Научные заметки на Исследование сопротивления по закону Ома V = IR Q = It расчеты GCSE руководство по физике примечания к закону Ома исследования сопротивления V = IR Q = It расчеты для школ, колледжей, академий, научных курсов, репетиторов, изображений рисунки-диаграммы для исследования сопротивления по закону Ома V = IR Q = Он вычисляет научные исправления примечания на Исследование сопротивления по закону Ома V = IR Q = It расчеты для пересмотра модулей физики примечания по темам физики, чтобы помочь в понимании Исследование сопротивления по закону Ома V = IR Q = It расчеты университетские курсы физики карьера в науке и физике вакансии в машиностроении технический лаборант стажировка инженер стажировка по физике США 8 класс 9 класс 10 AQA Заметки о пересмотре GCSE 9-1 по физике по закону Ома исследования сопротивления V = IR Q = It вычисления GCSE примечания к закону Ома исследования сопротивления V = IR Q = It расчеты Edexcel GCSE 9-1 физика наука примечания к редакции Исследование сопротивления по закону Ома V = IR Q = It расчеты для OCR GCSE 9-1 21 век научные заметки по физике об исследованиях сопротивления по закону Ома V = IR Q = Расчет OCR GCSE 9-1 Шлюз физики примечания к пересмотру исследований сопротивления по закону Ома V = IR Q = It вычисления WJEC gcse science CCEA / CEA gcse science

ВЕРХ СТРАНИЦЫ и субиндекс

Электродвижущая сила и внутреннее сопротивление

Электродвижущая сила (e) или e.м.ф. это энергия, обеспечиваемая элементом или батареей на кулон заряда, проходящего через них, она равна единицам измерения в вольтах (В). Это равно разности потенциалов на выводах ячейки, когда ток не течет.

  • e = электродвижущая сила в вольтах, В
  • E = энергия в джоулях, Дж
  • Q = заряд в кулонах, К

Батареи и элементы имеют внутреннее сопротивление (r) , что составляет Ом (Вт). Когда электричество течет по цепи, внутреннее сопротивление самого элемента сопротивляется прохождению тока, и поэтому тепловая (тепловая) энергия расходуется в самом элементе.

  • e = электродвижущая сила в вольтах, В
  • I = ток в амперах, А
  • R = сопротивление нагрузки в цепи в Ом, Вт
  • r = внутреннее сопротивление ячейки в Ом, Вт

Мы можем изменить приведенное выше уравнение;

, а затем на

В этом уравнении ( В, ) появляется разность потенциалов на клеммах , измеренная в вольтах (В). Это разность потенциалов на выводах ячейки, когда в цепи протекает ток, она всегда меньше, чем e.м.ф. ячейки.

Пример;

Q1) p.d. на выводах элемента составляет 3,0 В, когда он не подключен к цепи и не течет ток. Когда ячейка подключена к цепи и течет ток 0,37 А, клемма p.d. падает до 2,8 В. Какое внутреннее сопротивление ячейки?

График терминальных п.о. против нынешних

Если мы построим график зависимости разности потенциалов на клеммах (V) от тока в цепи (I), мы получим прямую линию с отрицательным градиентом.

Мы можем им переставить э.д.с. уравнение сверху, чтобы соответствовать общему выражению для прямой линии, y = mx + c.

Из красных прямоугольников выше видно;

  • пересечение по оси Y равно ЭДС. ячейки
  • градиент графика равен -r, где r — внутреннее сопротивление ячейки.
Сопротивление

и простые схемы — BCIT Physics 0312 Учебник

Сводка

  • Объясните происхождение закона Ома.
  • Рассчитывайте напряжения, токи или сопротивления по закону Ома.
  • Объясните, что такое омический материал.
  • Опишите простую схему.

Что движет током? Мы можем думать о различных устройствах, таких как батареи, генераторы, розетки и т. Д., Которые необходимы для поддержания тока. Все такие устройства создают разность потенциалов и условно называются источниками напряжения. Когда источник напряжения подключен к проводнику, он применяет разность потенциалов [латекс] \ boldsymbol {V} [/ latex], которая создает электрическое поле.Электрическое поле, в свою очередь, воздействует на заряды, вызывая ток.

Ток, протекающий через большинство веществ, прямо пропорционален приложенному к нему напряжению [латекс] \ boldsymbol {V} [/ latex]. Немецкий физик Георг Симон Ом (1787–1854) первым экспериментально продемонстрировал, что ток в металлической проволоке прямо пропорционален приложенному напряжению :

[латекс] \ boldsymbol {I \ propto V}. [/ Латекс]

Это важное соотношение известно как закон Ома.Его можно рассматривать как причинно-следственную связь, в которой напряжение является причиной, а ток — следствием. Это эмпирический закон, подобный закону трения — явление, наблюдаемое экспериментально. Такая линейная зависимость возникает не всегда.

Если напряжение управляет током, что ему мешает? Электрическое свойство, препятствующее току (примерно такое же, как трение и сопротивление воздуха), называется сопротивлением RR размером 12 {R} {}. Столкновения движущихся зарядов с атомами и молекулами вещества передают энергию веществу и ограничивают ток.Сопротивление обратно пропорционально току, или

.

[латекс] \ boldsymbol {I \ propto} [/ latex] [латекс] \ boldsymbol {\ frac {1} {R}}. [/ Latex]

Таким образом, например, ток уменьшается вдвое, если сопротивление увеличивается вдвое. Комбинируя отношения тока к напряжению и тока к сопротивлению, получаем

[латекс] \ boldsymbol {I =} [/ latex] [латекс] \ boldsymbol {\ frac {V} {R}}. [/ Latex]

Это соотношение также называется законом Ома. Закон Ома в такой форме действительно определяет сопротивление определенных материалов.Закон Ома (как и закон Гука) не универсален. Многие вещества, для которых действует закон Ома, называются омическими. К ним относятся хорошие проводники, такие как медь и алюминий, и некоторые плохие проводники при определенных обстоятельствах. Омические материалы имеют сопротивление [латекс] \ boldsymbol {R} [/ latex], которое не зависит от напряжения [латекс] \ boldsymbol {V} [/ latex] и тока [латекс] \ boldsymbol {I} [/ latex]. Объект с простым сопротивлением называется резистором , даже если его сопротивление невелико.Единицей измерения сопротивления является Ом и обозначается символом [латекс] \ Омега [/ латекс] (греческое омега в верхнем регистре). Перестановка [latex] \ boldsymbol {I = V / R} [/ latex] дает [latex] \ boldsymbol {R = V / I} [/ latex], поэтому единицы сопротивления равны 1 Ом = 1 вольт на ампер:

[латекс] \ boldsymbol {1 \; \ Omega = 1} [/ latex] [латекс] \ boldsymbol {\ frac {V} {A}} [/ латекс]

На рисунке 1 показана схема простой схемы. Простая схема имеет один источник напряжения и один резистор. Можно предположить, что провода, соединяющие источник напряжения с резистором, имеют незначительное сопротивление, или их сопротивление можно указать в [латексе] \ boldsymbol {R} [/ латексе].

Рис. 1. Простая электрическая цепь, в которой замкнутый путь прохождения тока обеспечивается проводниками (обычно металлическими), соединяющими нагрузку с выводами батареи, представленными красными параллельными линиями. Зигзагообразный символ представляет собой единственный резистор и включает любое сопротивление в соединениях с источником напряжения.

Пример 1: Расчет сопротивления: автомобильная фара

Какое сопротивление проходит у автомобильной фары? 2.50 А течет при подаче на него 12,0 В?

Стратегия

Мы можем изменить закон Ома, как указано в [latex] \ boldsymbol {I = V / R} [/ latex], и использовать его для определения сопротивления.

Решение

Перестановка [latex] \ boldsymbol {I = V / R} [/ latex] и замена известных значений дает

[латекс] \ boldsymbol {R =} [/ latex] [латекс] \ boldsymbol {\ frac {V} {I}} [/ latex] [латекс] \ boldsymbol {=} [/ latex] [латекс] \ boldsymbol {\ frac {12. 12 \; \ Omega} [/ latex] или более.{-5} \; \ Omega} [/ latex], а сверхпроводники вообще не имеют сопротивления (они неомичны). Сопротивление связано с формой объекта и материалом, из которого он состоит, как будет показано в главе 20.3 Сопротивление и удельное сопротивление.

Дополнительную информацию можно получить, решив [latex] \ boldsymbol {I = V / R} [/ latex], что дает

[латекс] \ boldsymbol {V = IR}. [/ Latex]

Это выражение для [latex] \ boldsymbol {V} [/ latex] можно интерпретировать как падение напряжения на резисторе, вызванное протеканием тока [latex] \ boldsymbol {I} [/ latex].Для обозначения этого напряжения часто используется фраза [latex] \ boldsymbol {IR} [/ latex] drop . Например, у фары в Примере 1 падение [latex] \ boldsymbol {IR} [/ latex] составляет 12,0 В. Если напряжение измеряется в различных точках цепи, будет видно, что оно увеличивается на источнике напряжения и уменьшается. на резисторе. Напряжение аналогично давлению жидкости. Источник напряжения подобен насосу, создающему перепад давления, вызывая ток — поток заряда. Резистор похож на трубу, которая снижает давление и ограничивает поток из-за своего сопротивления.Здесь сохранение энергии имеет важные последствия. Источник напряжения подает энергию (вызывая электрическое поле и ток), а резистор преобразует ее в другую форму (например, тепловую энергию). В простой схеме (с одним простым резистором) напряжение, подаваемое источником, равно падению напряжения на резисторе, поскольку [latex] \ boldsymbol {PE = q \ Delta V} [/ latex], и то же самое qq размер 12 {q} {} проходит через каждую. Таким образом, энергия, подаваемая источником напряжения, и энергия, преобразуемая резистором, равны.(См. Рисунок 2.)

Рис. 2. Падение напряжения на резисторе в простой цепи равно выходному напряжению батареи.

Установление соединений: сохранение энергии

В простой электрической цепи единственный резистор преобразует энергию, поступающую от источника, в другую форму. Здесь о сохранении энергии свидетельствует тот факт, что вся энергия, подаваемая источником, преобразуется в другую форму одним резистором. Мы обнаружим, что сохранение энергии имеет другие важные применения в схемах и является мощным инструментом анализа схем.

Исследования PhET: закон Ома

Посмотрите, как уравнение закона Ома соотносится с простой схемой. Отрегулируйте напряжение и сопротивление и посмотрите, как изменяется ток по закону Ома. Размеры символов в уравнении изменяются в соответствии с принципиальной схемой.

Рисунок 3. Закон Ома
  • Простая схема — это схема , в которой есть один источник напряжения и одно сопротивление.
  • Одно из утверждений закона Ома устанавливает связь между током [латекс] \ boldsymbol {I} [/ latex], напряжением [латекс] \ boldsymbol {V} [/ latex] и сопротивлением [латекс] \ boldsymbol {R} [/ latex] в простой схеме быть [latex] \ boldsymbol {I = \ frac {V} {R}} [/ latex].
  • Сопротивление выражается в единицах Ом ([латекс] \ boldsymbol {\ Omega} [/ latex]), относящихся к вольтам и амперам с помощью [латекса] \ boldsymbol {1 \; \ Omega = 1 \; \ textbf {V} / \ textbf {A}} [/ латекс].
  • Падение напряжения или [латекс] \ boldsymbol {IR} [/ latex] на резисторе, вызванное протекающим через него током, определяемое выражением [latex] \ boldsymbol {V = IR} [/ latex].

Концептуальные вопросы

1: Падение [латекс] \ boldsymbol {IR} [/ latex] на резисторе означает изменение потенциала или напряжения на резисторе.Изменится ли ток при прохождении через резистор? Объяснять.

2: Как падение [латекс] \ boldsymbol {IR} [/ latex] в резисторе похоже на падение давления в жидкости, протекающей по трубе?

Задачи и упражнения

1: Какой ток протекает через лампочку фонаря на 3,00 В, когда его горячее сопротивление составляет [латекс] \ boldsymbol {3.60 \; \ Omega} [/ latex]?

2: Вычислите эффективное сопротивление карманного калькулятора с 1.Аккумулятор 35 В и через который протекает 0.200 мА.

3: Каково эффективное сопротивление стартера автомобиля, когда через него проходит 150 А, когда аккумулятор автомобиля подает на двигатель 11,0 В?

4: Сколько вольт подается для работы светового индикатора DVD-плеера с сопротивлением [латекс] \ boldsymbol {140 \; \ Omega} [/ latex], если через него проходит 25,0 мА?

5: (a) Найдите падение напряжения в удлинителе с символом [латекс] \ bold {0.0600- \ Omega} [/ latex] сопротивление, через которое протекает ток 5,00 А. (б) В более дешевом шнуре используется более тонкая проволока, и его сопротивление составляет [латекс] \ boldsymbol {0,300 \; \ Omega} [/ латекс]. Какое в нем падение напряжения при протекании 5.00 А? (c) Почему напряжение на любом используемом приборе снижается на эту величину? Как это повлияет на прибор?

6: ЛЭП подвешена к металлическим опорам со стеклянными изоляторами, имеющими сопротивление [латекс] \ boldsymbol {1,00 \ times 10 ^ 9 \; \ Omega} [/ латекс].Какой ток протекает через изолятор при напряжении 200 кВ? (Некоторые линии высокого напряжения — постоянного тока.)

Глоссарий

Закон Ома
— эмпирическое соотношение, указывающее, что ток I пропорционален разности потенциалов В , В ; его часто записывают как I = V / R , где R — сопротивление
сопротивление
электрическое свойство, препятствующее току; для омических материалов это отношение напряжения к току, R = V / I
Ом
единица сопротивления, равная 1Ω = 1 В / А
омическое
вид материала, для которого действует закон Ома
простая схема
схема с одним источником напряжения и одним резистором

Решения

Задачи и упражнения

1: 0.{-2} \; \ Omega} [/ латекс]

5: (а) 0,300 В

(б) 1,50 В

(c) Напряжение, подаваемое на любой используемый прибор, снижается, поскольку общее падение напряжения от стены до конечного выхода прибора является фиксированным. Таким образом, если падение напряжения на удлинителе велико, падение напряжения на приборе значительно уменьшается, поэтому выходная мощность прибора может быть значительно уменьшена, что снижает способность прибора работать должным образом.

Калькулятор параллельного и последовательного сопротивления

Параллельный и последовательный


Сопротивление
Параллельное сопротивление XXXX Ом
Сопротивление серии XXXX Ом


Рассчитать Прозрачный
⚠️ Сообщить о проблеме

Сопротивление — это электрический элемент, который не позволяет легко протекать через него току. В нашей повседневной жизни сопротивление используется во многих электрических цепях и устройствах.Внутри этих электрических устройств сопротивления расположены в различных конфигурациях.

Вы также, должно быть, наблюдали, выполняя соединения на макетной плате в своей лаборатории электроники, как резисторы подключаются по-разному. В зависимости от способа соединения их концов различают два основных типа цепей сопротивления:

    Цепи серии
  • Параллельные схемы

По мере продвижения в этом посте мы поймем работу, сравнение, вычисления и различия между этими последовательными и параллельными цепями.

Схемы серии

Два или более резистора считаются подключенными последовательно, если ток, протекающий через них, одинаков. Другими словами, ток, исходящий от источника, или общий ток не будут разветвляться по какому-либо другому пути, а будут двигаться только по одному прямому пути.

При последовательном соединении сопротивлений чистое сопротивление цепи является суммой всех сопротивлений цепи.

На следующей принципиальной схеме показаны три последовательно соединенных сопротивления R 1 , R 2 и R 3 .Общее сопротивление этой последовательной цепи составляет R нетто = 1 + 2 + 3 .

Как правило, если «n» сопротивлений соединены последовательно, общее сопротивление составляет R нетто = R 1 + R 2 +…. R .


Например, если в цепи есть три последовательно подключенных резистора по 10 Ом, и источник напряжения 30 В, то ток, протекающий в цепи и через каждый из них, определяется выражением I = V / R = 10/30 = 0. .33 А.

Параллельные цепи

Два или более резистора считаются подключенными параллельно, если они соединены головками на одной стороне и выводами на другой стороне цепи. В параллельной цепи ток, исходящий от источника, или полный ток будет ветвиться в месте соединения, где головки сопротивлений встречаются, а затем течет в разном количестве в каждом резисторе, а затем снова объединяется в точке встречи хвостов резисторы и течет в сторону начала координат.

При параллельном соединении сопротивлений сопротивление цепи, обратное сопротивлению цепи, является суммой значений, обратных сопротивлению всех сопротивлений цепи.

На следующей принципиальной схеме показаны три параллельно включенных сопротивления R 1 , R 2 и R 3 . Общее сопротивление этой параллельной цепи составляет 1 / R net = 1 / R = 1 / R 1 + 1 / R 2 + 1 / R 3 .


Как правило, если «n» сопротивлений соединены параллельно, общее сопротивление составляет:

1 / R net = 1 / R = 1 / R + 1 / R 2 + 1 / R 3 +….+ 1 / R n

Например, если три резистора 4 Ом, 8 Ом и 8 Ом соединены параллельно в цепи с питанием 10 В, то общее сопротивление цепи определяется как: 1 / Rnet = 1/4 + 1/8 + 1/8 = ½ или Rnet = 2 Ом

Тогда ток, протекающий по цепи, равен V / I = 10/2 = 5 ампер.

Если вы хотите рассчитать ток в каждом резисторе, вы можете использовать закон Ома:

Ток через резистор 4 Ом I 1 = 10/4 = 2.5 А

Ток через резистор 8 Ом I 1 = 10/8 = 1,25 A

Ток через резистор 4 Ом I 1 = 10/8 = 1,25 A

Обратите внимание, что если вы сложите отдельные токи через каждый резистор, вы получите общий ток, протекающий в цепи.

Комбинация последовательных и параллельных резисторов

Если вы найдете схему, в которой резисторы включены последовательно и параллельно, то вам нужно решить комбинацию резисторов шаг за шагом, учитывая, находятся ли они последовательно или параллельно с соседними, и затем прийти к окончательному сопротивлению цепи.

Пункты о последовательном и параллельном соединении резисторов:

  • Эффективное сопротивление последовательной цепи всегда больше, чем сопротивление каждого резистора в цепи.
  • Эффективное сопротивление параллельной цепи всегда меньше, чем сопротивление каждого резистора в цепи.
  • Ток в каждом из последовательно соединенных резисторов одинаков, а напряжение на каждом параллельном резисторе одинаково.
  • В цепи последовательных резисторов, если одно сопротивление повреждается, вся цепь разрывается и ведет себя как разомкнутая цепь.
  • В схеме параллельных резисторов, если один резистор поврежден, ток продолжает течь в других резисторах, и схема будет продолжать работать, но с другим значением сопротивления цепи.

Как вам помогает калькулятор последовательного и параллельного сопротивления CalculatorHut?

В нашей повседневной жизни мы сталкиваемся с последовательным и параллельным сочетанием резисторов во многих местах. Например, для цепей освещения мы используем параллельное соединение, а для приборов, которые работают непосредственно от сети, дается последовательное соединение.

CalculatorHut, универсальный центр научных и ненаучных онлайн-калькуляторов, предлагает бесплатный онлайн-калькулятор последовательного и параллельного сопротивления, который решит все ваши потребности в онлайн-калькуляторе бесплатно. Вы можете рассчитать до десяти резисторов, которые подключены последовательно или параллельно, с помощью этого удобного онлайн-калькулятора последовательного и параллельного сопротивления. Это очень удобный инструмент для студентов, который помогает им проверить правильность расчетов сопротивления.

CalculatorHut также предлагает широкий выбор более 100 калькуляторов по различным темам — калькуляторы здоровья, финансовые калькуляторы, калькуляторы транспортных средств, физические калькуляторы, химические калькуляторы, математические калькуляторы и многие другие бесплатные онлайн-калькуляторы для научных расчетов.

Наши читатели также могут получить бесплатный виджет любого калькулятора из нашего широкого диапазона калькуляторов для встраивания в качестве виджетов на свои веб-сайты. Для этого они могут написать нам письмо по адресу [email protected].

Мы пропустили какой-нибудь бесплатный онлайн-калькулятор? Пожалуйста дай нам знать.Мы будем более чем счастливы удовлетворить ваши потребности в бесплатном онлайн-калькуляторе бесплатно и всегда!

Вот еще одна фантастическая новость! Вы можете бесплатно носить с собой в кармане наш широкий ассортимент онлайн-калькуляторов. Да! Бесплатное приложение CalculatorHut — ваш друг, который упрощает и упрощает любые вычисления! Удачных расчетов!

бесплатных онлайн-курсов по математике, естественным наукам и инженерным наукам

FutureLearn предлагает курсы естествознания, инженерии и математики, охватывающие широкий и увлекательный круг предметов.

Поскольку это одна из самых популярных и быстро развивающихся областей знаний, никогда не бывает плохого времени, чтобы освежить основы или изучить некоторые специальные области.

Преимущества курсов STEM

Люди со знанием предметов STEM сейчас очень востребованы.

Многие технологические достижения требуют хотя бы некоторого понимания науки, техники и математических принципов. Итак, если вы относительно новичок в этой области знаний, прохождение онлайн-курса может направить вас на верный путь.

Если вы всегда хотели исследовать эти интересные области, онлайн-обучение позволит вам действовать в удобном для вас темпе. С относительно короткими курсами, часто требующими всего несколько часов обучения каждую неделю, вы скоро сможете овладеть некоторыми принципами этих предметов STEM.

Для тех, кто уже работает в отрасли, чтобы быть в курсе текущих тенденций, требуются определенные усилия. С нашими онлайн-курсами по естествознанию, инженерии и математике вы можете продолжить структурированное развитие своих знаний.

Изучение естественных наук, инженерии и математики

Чтобы дать нашим учащимся широкий выбор областей обучения, мы собрали курсы по разным темам. Сюда входят такие широкие области, как математика, физика, химия и инженерия. Есть также предметы в более специализированных областях, таких как наука о данных и науки о Земле.

Как видите, есть так много интересных мест для изучения. Если вы ищете введение в математику или более подробный обзор биологии и биотехнологии, вы можете найти учебные материалы, соответствующие вашему уровню и потребностям.

Выберите подходящий вам формат курса

Помимо множества коротких онлайн-курсов, мы также предлагаем несколько более подробных вариантов. Итак, если у вас уже есть понимание некоторых основ, вы можете развить эти знания еще дальше.

С нашими программами вы можете пройти несколько специализированных курсов. Благодаря этому подробному обзору вы можете улучшить свое понимание и даже получить академическую аккредитацию.

Вы также найдете микропрограммные данные.Эти возможности обучения дают вам возможность повысить квалификацию в новой области, одновременно получая за это академический балл.

Наконец, вы также можете изучить наши онлайн-степени по естествознанию, инженерии и математике. Эти курсы, предлагаемые ведущими университетами, могут быть очными или заочными.

Mr Toogood Physics — Резисторы и сопротивление

Сопротивление

В большинстве цепей ток течет из-за движения свободных электронов по проводнику.SOme-проводники имеют больше свободных электронов, чем другие, и, следовательно, являются лучшими проводниками, чем проводники с меньшим количеством свободных электронов. Материалы без свободных электронов называются изоляторами и не пропускают через них ток. Есть также материалы, которые находятся где-то посередине и будут только высвобождать электроны или пропускать ток в ответ на внешний стимул, такой как свет, тепло или электрическое поле. Эти материалы называются полупроводниками и являются строительными блоками современной цифровой технологии.

В любом металле атомы выделяют электроны, образуя кристаллическую решетку, и эти свободные электроны беспорядочно перемещаются между ионными остовами атомов металла. Обычно, поскольку движение электронов является случайным, их чистая скорость равна нулю. Однако, когда ЭДС помещается поперек проводника, электроны будут двигаться от отрицательного вывода к положительному выводу.

Рисунок 1: Если бы мы могли видеть внутри провода …

Когда электроны начинают двигаться, их беспорядочное движение нарушается, но поскольку существует огромное количество ионных ядер, преграждающих путь между выводами, между ними происходит много столкновений. и электроны.Это приводит к тому, что траектория электронов не прямая, а общая скорость остается довольно низкой, однако теперь чистая скорость дрейфа меняется от отрицательной к положительной. Эта серия столкновений уменьшает количество энергии, которую имеют электроны, происходит передача электрической потенциальной энергии тепловой энергии. В более плохих проводниках больше столкновений и передается большее количество электрического потенциала. Это называется сопротивлением. Чем больше столкновений, тем большее количество энергии передается теплу, тем больше разность потенциалов и тем больше сопротивление компонента.

Резисторы — это компоненты, которые имеют фиксированное значение сопротивления в диапазоне разностей потенциалов. Ток через них прямо пропорционален разности потенциалов на них. Говорят, что они следуют закону Ома, который описан в:

$$ R = \ frac {V} {I} $$

Закон

Ома гласит, что ток через проводник прямо пропорционален разности потенциалов на нем для проводника при постоянной температуре. Проводники, такие как резисторы, ведут себя омически при фиксированных температурах, и график зависимости тока от разности потенциалов дает прямую линию, проходящую через начало координат.Сопротивление компонента обратно градиенту. Резистор с высоким сопротивлением будет иметь относительно неглубокую линию, тогда как компонент с низким сопротивлением будет иметь крутую линию. Глядя на графики ниже, мы видим, что при одинаковом увеличении p.d. синяя линия имеет меньшее увеличение тока, протекающего через нее, чем красная линия. Другие компоненты имеют отличительные формы для графиков IV , о которых вам необходимо знать.

Рисунок 2: I.V. Характеристики различных компонентов.

Лампа накаливания или старомодная лампочка работает, пропуская большой ток через тонкий кусок проволоки, который становится очень горячим и начинает раскаливаться докрасна. Это возбуждает инертный аргон внутри колбы, чтобы произвести свет. При низких значениях p.d. лампочка с проводом ведет себя омически и линия прямая, но когда п.о. увеличивается сопротивление лампы увеличивается, и линия сглаживается.

Диод является полупроводником и пропускает ток только при определенных условиях.В случае поляризованных диодов ток может течь только в одном направлении, поэтому, если разность потенциалов обратная, ток не может течь, что объясняет горизонтальную линию в отрицательном квадранте графика. Когда они размещаются в прямом направлении, они позволяют току легко течь, но только после достижения порогового напряжения, после этого их сопротивление падает почти до нуля, и линия поднимается почти вертикально.

Наверх


Сопротивление и температура

Когда проводник нагревается, энергия передается частицам внутри проводника, и они начинают двигаться быстрее.В твердом теле, поскольку частицы находятся в фиксированном положении, это означает, что они начинают вибрировать быстрее. Это приводит к увеличению числа столкновений между частицами твердого проводника и электронами. Фактически, длина свободного пробега электронов уменьшается, и количество заряда, протекающего в секунду, также уменьшается, что приводит к увеличению сопротивления.

Нагрев может вызвать увеличение числа столкновений между электронами и частицами, и обратное также верно.Увеличение тока через проводник увеличит количество столкновений между частицами и электронами. Это вызывает передачу энергии от электронов к частицам внутри проводника и генерирует тепло. Это, в свою очередь, увеличивает сопротивление проводника. Этот эффект можно отчетливо увидеть, когда лампа накаливания включена и ток, протекающий через нее, анализируется в течение первых нескольких секунд. График ниже был получен с использованием регистратора данных в классе.

Рисунок 3: Как изменяется ток через лампочку накаливания в первые 3 секунды после ее включения.

При включении лампочки внезапно увеличивается ток, протекающий через лампочку. Поскольку колба остыла перед включением, частицы в нити накала не очень быстро вибрируют, и колба имеет низкое сопротивление. Это означает, что через него может протекать большой ток. По мере протекания тока электроны сталкиваются с ионами в нити накала и заставляют ее нагреваться, увеличивая сопротивление колбы. Это более высокое сопротивление означает, что может протекать меньший ток, и мы видим, что ток снижается до более низкого постоянного значения, известного как рабочий ток.Это хороший пример отрицательной обратной связи , где выходной сигнал системы в момент времени a приводит к тому, что выходной сигнал в момент времени b становится ниже. Это объясняет форму ВАХ лампы накаливания и то, почему сопротивление увеличивается с увеличением силы тока.

По мере увеличения тока через колбу накаливания увеличивается и температура нити накала, и, следовательно, увеличивается сопротивление. Это дает отчетливую S-образную кривую.

Взаимосвязь между сопротивлением и температурой может быть использована для создания точных термометров, которые можно использовать в широком диапазоне температур.Вы проведете в классе эксперимент по созданию простейшего термометра с проводом сопротивления.

Наверх


Резисторы по цепи

Когда в цепи несколько резисторов, может потребоваться объединить их значения, чтобы вычислить общее сопротивление или ток, протекающий в цепи. Когда все резисторы включены последовательно, это довольно просто, поскольку значения резисторов можно просто сложить.В приведенном ниже примере есть три резистора, каждый с разным сопротивлением.

Рисунок 4: Добавление резисторов в последовательную цепь.

Суммарное сопротивление для этой цепи:

$$ \ large R_ {Total} = R_ {1} + R_ {2} + R_ {3} + \ cdots $$ $$ \ amount {10} {Ω} + \ amount {5} {Ω} + \ amount {12} {Ω} = \ amount {27} {Ω}; $$

Несмотря на простую взаимосвязь, выражение для полного сопротивления в последовательной цепи может быть получено, если вспомнить, что сумма разностей потенциалов на каждом резисторе равна ЭДС, подаваемой в цепь (2-й закон Кирхгофа).

\ begin {align} ε & = V_ {1} + V_ {2} + V_ {3} + \ cdots \\ \\ IR_ {T} & = IR_ {1} + IR_ {2} + IR_ {3} + \ cdots \\ \\ IR_ {T} & = I \ left (R_ {1} + R_ {2} + R_ {3} + \ cdots \ right) \\ \\ R_ {T} & = R_ {1} + R_ {2} + R_ {3} + \ cdots \ end {align}

Наверх


Резисторы параллельно

Для резисторов, включенных параллельно, ситуация несколько сложнее, и стоит подумать об этом простыми словами. Разница между последовательными и параллельными цепями заключается в том, что ток разделяется на соединениях в параллельной цепи.Каждая единица заряда получает от клетки определенное количество энергии. Он будет использовать эту энергию только при прохождении через сопротивление. На диаграмме ниже каждая оранжевая точка представляет $ \ amount {1} {C} $ заряда, несущего $ \ amount {1} {J} $ электрического потенциала. На перекрестке заряды либо повернут направо, либо продолжат движение прямо. Поскольку сопротивление каждой ветви одинаково, через каждую проходит одинаковое количество тока. Каждый заряд выполняет $ \ amount {1} {J} $ работы, когда проходит через сопротивление, а это означает, что разность потенциалов на каждой ветви равна $ \ amount {1} {V} $.

Рисунок 5: Разделение тока на стыке.

Мы также можем видеть, что ток на каждой ветви меньше, чем ток до соединения, но количество энергии, которое имеет каждый кулон заряда, одинаково до и после соединения. Мы также можем применить к этой схеме два закона Кирхгофа:

  • Ток в каждой ветви меньше, чем ток до соединения, но их сумма равна току до соединения. Это удовлетворяет первому закону.
  • Принимая каждую ветвь за отдельную петлю, мы можем видеть, что разность потенциалов на каждой ветке такая же, как энергия, удерживаемая каждым кулоновским зарядом (ЭДС) перед переходом, что, согласно второму закону, должно быть кейс.

Мы можем применить 1-й закон Кирхгофа, чтобы вывести уравнение для полного сопротивления в параллельной цепи. Сумма токов на каждой ветви будет равна общему току на выходе из ячейки:

\ begin {align} I_ {T} & = I_ {1} + I_ {2} + I_ {3} + \ cdots \\ \\ \ frac {V} {R_ {T}} & = \ frac {V} {R_ {1}} + \ frac {V} {R_ {2}} + \ frac {V} {R_ {3}} + \ cdots \\ \\ \ frac {1} {R_ {T}} & = \ frac {1} {R_ {1}} + \ frac {1} {R_ {2}} + \ frac {1} {R_ {3}} + \ компакт-диски \ end {align}

Это уравнение заслуживает некоторого обсуждения, так как оно приводит к несколько нелогичному выводу, что если мы добавим больше параллельных резисторов, общее сопротивление схемы фактически уменьшится

Если мы посмотрим на схему ниже, то увидим, что с одним резистором $ \ amount {2} {Ω} $ разность потенциалов будет $ \ amount {6} {V} $, а ток будет $ \ amount {3} {A} $.

Рисунок 6: Ток, потребляемый одним резистором

Если второй резистор большего номинала добавляется параллельно первому, открывается другой путь или путь, через который протекает ток. Даже из-за того, что сопротивление на этом пути выше, через него протекает меньше тока, теперь через цепь протекает больше тока в целом. Мы можем рассчитать ток в каждой ветви, используя закон Ома, помня, что каждая ветвь будет иметь одинаковую разность потенциалов.

Рисунок 7: Ток, потребляемый двумя параллельными резисторами.

Для резистора R 1 :

\ begin {align} I & = \ frac {V} {R} \\ \\ I & = \ frac {\ amount {6.0} {V}} {\ amount {2.0} {Ω}} \\ \\ I & = \ amount {3.0} {A} \ end {align}

Для резистора R 2 :

\ begin {align} I & = \ frac {V} {R} \\ \\ I & = \ frac {\ amount {6.0} {V}} {\ amount {20} {Ω}} \\ \\ I & = \ amount {0.30} {A} \ end {align}

Суммарный ток складывается из токов на ответвлениях:

$$ I_ {Итого} = \ количество {3.0} {A} + \ amount {0.30} {A} = \ amount {3.3} {A} $$

Когда добавляется третий резистор с еще более высоким значением, чем любой из первых двух, открывается еще один путь для прохождения тока, и ток, выходящий из ячейки, снова увеличивается.

Рисунок 8: Ток, потребляемый тремя параллельными резисторами.

Поскольку каждый раз из ячейки выходит больше тока и ячейка имеет фиксированную ЭДС, следует сделать вывод, что поскольку $ R = \ frac {V} {I} $, поскольку I увеличивается каждый раз, когда добавляется резистор, то общее сопротивление цепи должно уменьшиться.

Мы можем рассчитать сопротивление конечной цепи, используя уравнение, которое мы вывели выше:

\ begin {align} \ frac {1} {R_ {T}} & = \ frac {1} {R_ {1}} + \ frac {1} {R_ {2}} + \ frac {1} {R_ {3}} \\ \\ \ frac {1} {R_ {T}} & = \ frac {1} {\ amount {2.0} {Ω}} + \ frac {1} {\ amount {20} {Ω}} + \ frac {1} {\ amount {200} {Ω}} \\ \\ \ frac {1} {R_ {T}} & = 0,555 \ end {align}

Мы должны быть очень осторожны, так как это значение, которое мы только что вычислили, это $ \ frac {1} {R} $, а не $ R $, поэтому мы должны найти обратное этому значению, чтобы найти сопротивление.

$$ R = \ frac {1} {0.555} = \ amount {1.8} {Ω} $$

Обратите внимание, что полное сопротивление цепи на самом деле ниже, чем у наименьшего резистора в цепи. Вы можете использовать это как хороший способ проверить свои расчеты, поскольку сопротивление параллельной сети всегда будет ниже, чем сопротивление любого из резисторов в этой сети.


Комбинированные резисторы последовательно и параллельно

Когда мы сталкиваемся с более сложными схемами, которые имеют комбинации последовательно соединенных резисторов и , параллельных параллели, мы должны следовать следующим правилам, чтобы найти полное сопротивление цепи и, таким образом, произвести дальнейшие вычисления:

  1. Работайте от самой дальней от ячейки ответвления и выполняйте вычисления по направлению к ячейке.
  2. Добавьте все последовательные резисторы, которые находятся на параллельных ветвях.
  3. Рассчитайте сопротивление параллельных сетей.
  4. Добавьте это сопротивление к любым другим последовательным резисторам.

В приведенном ниже примере ветвь, содержащая R 4 и R 5 , должна обрабатываться первой, поскольку она наиболее удалена от ячейки.

Рисунок 9: Объединение резисторов последовательно и параллельно.

$$ R_ {4} + R_ {5} = \ amount {8.0} {Ω} + \ amount {6.0} {Ω} = \ amount {14} {Ω} $$

Таким образом, эта ветвь имеет сопротивление $ \ amount {14} {Ω} $, которое можно объединить с R 3 со значением $ \ amount {12} {Ω} $, чтобы найти полное сопротивление параллельной сети. . Мы знаем, что полное сопротивление этой части схемы будет меньше $ \ amount {12} {Ω} $, так что это позволит нам проверить наш ответ на ошибки.

\ begin {align} \ frac {1} {R_ {T}} & = \ frac {1} {R_ {3}} + \ frac {1} {R_ {4 + 5}} \\ \\ \ frac {1} {R_ {T}} & = \ frac {1} {\ amount {12} {Ω}} + \ frac {1} {\ amount {14} {Ω}} \\ \\ \ frac {1} {R_ {T}} & = 0.1547619 \\ \\ R_ {T} & = \ amount {6.5} {Ω} \ end {align}

Наконец, теперь мы можем добавить это сопротивление к двум последовательным резисторам, R 1 и R 2 , чтобы найти общее сопротивление цепи,

.

$$ \ amount {5.0} {Ω} + \ amount {2.0} {Ω} + \ amount {6.5} {Ω} = \ amount {13.5} {Ω} $$

Теперь мы знаем полное сопротивление цепи, учитывая, что ЭДС батареи равна $ \ amount {3} {V} $, мы можем вычислить ток как $ \ amount {0.22} {A}

долл. США

Наверх


Рабочий пример

В этом примере вопрос начинается с некоторых расчетов сопротивления, прежде чем вас попросят сравнить потенциалы между отмеченными точками.

На принципиальной схеме ниже показана батарея $ \ amount {12} {V} $ с незначительным внутренним сопротивлением, подключенная к комбинации из трех резисторов и термистора.

Рисунок 10: Рабочий пример 1.
  1. Когда сопротивление термистора равно $ \ amount {5.0} {kΩ}
  2. долл. США
    1. рассчитать полное сопротивление цепи,
    2. Поскольку обе ветви имеют по два резистора, нам нужно сложить их вместе, чтобы найти общее сопротивление на каждой ветви, прежде чем мы найдем сопротивление параллельной сети.

      Сопротивление между A-E

      $$ \ amount {20} {kΩ} + \ amount {20} {kΩ} = \ amount {40} {kΩ} $$

      Сопротивление между B-F

      $$ \ amount {10} {kΩ} + \ amount {5.0} {kΩ} = \ amount {15} {kΩ} $$

      Теперь, когда мы знаем полное сопротивление каждой ветви, мы можем использовать уравнение для параллельных резисторов, чтобы найти полное сопротивление цепи.

      \ begin {align} \ frac {1} {R_ {T}} & = \ frac {1} {\ amount {40} {kΩ}} + \ frac {1} {\ amount {15} {kΩ}} \\ \ frac {1} {R_ {T}} & = 0,16 \\ \\ R_ {T} & = \ amount {10.9} {kΩ} = \ amount {11} {kΩ} \ end {align}

      При выполнении подобных вычислений с промежуточным вычислением, в данном случае для $ \ frac {1} {R_ {T}} $, вы должны либо записать полный ответ, как показано на вашем калькуляторе, либо использовать память калькулятора. настройки, чтобы сохранить его для последнего шага.Для окончательного ответа я округлил результат до того же количества значащих цифр, что и данные в вопросе.


    3. вычислить ток в батарее (в $ \ units {mA} $).
    4. С сопротивлением цепи, рассчитанным в части i. и ЭДС ячейки, указанной в вопросе, мы можем использовать закон Ома, чтобы найти полный ток, протекающий в цепи:

      \ begin {align} I & = \ frac {V} {R} \\ \\ I & = \ frac {\ quantity {12} {V}} {\ amount {10.9} {кОм}} \\ \\ I & = \ amount {1.10} {mA} \ end {align}

      Поскольку все сопротивления выражены в $ \ units {kΩ} $, а вопрос задает ток в $ \ units {mA} $, нет необходимости преобразовывать сопротивления в $ \ units {Ω} $.


  3. Вольтметр с высоким сопротивлением используется для измерения разности потенциалов (pd) между точками A-C, D-F и C-D по очереди.
    Заполните следующую таблицу, указав показания вольтметра в каждой из трех позиций.

  4. положение вольтметра стр. / V
    А-С
    Д-Ф
    К-Д

    Разницу потенциалов между A-C и D-F легко вычислить без дополнительных вычислений. Мы знаем, что п.д. на каждой ветви находится $ \ amount {12} {V} $, а на первой ветви два резистора имеют одинаковое значение, поэтому p.d. в каждом из них равны, в данном случае $ \ amount {6} {V} $.

    Сопротивление резистора между ответвлением в два раза больше сопротивления термистора между D-F, поэтому p.d. через резистор будет вдвое больше p.d. поперек термистора, поэтому p.d. поперек термистора находится $ \ amount {4} {V} $.

    Наконец, между двумя ветвями подключается вольтметр, как показано на схеме ниже:

    Рисунок 11: Рабочий пример 2.

    Поначалу кажется, что решить эту проблему очень сложно, но, применив 2-й закон Кирхгофа к каждой ветви и сравнив величину электрического потенциала, удерживаемого током после прохождения через первый резистор, мы можем можно сравнить разность потенциалов в двух ветвях.

    В верхней ветви ток содержит $ \ amount {12} {V} — \ amount {6} {V} = \ amount {6} {V} $, а в нижней ветви мы уже сделали вывод, что ток имеет $ \ amount {4} {V} $.

Добавить комментарий

Ваш адрес email не будет опубликован. Обязательные поля помечены *